You are on page 1of 128

Balancing Moments

Forces and Rotation


We can make an object rotate by applying one force or a system of forces to an object. For
example, when you push a door, the door will turn on its hinges.

The turning effect of the force causes the object to rotate.

Examples of Forces Causing Rotation

Balancing Moments

Moments
 The turning effect of a force is called the moment. Moments are used to describe the turning effect
of a force. Previously, we mentioned that the turning effect causes the rotation of an object. Now, we
can say that moments cause the rotation of an object.
 Moments vary in size. The size of the moment will affect the amount of rotation produced. If a
moment is bigger, then the object will experience a lot of rotation. If the moment is small, then the
object will experience very little turning force. In the next section, we will discuss how to change the
size of a moment.
Balancing Moments

Calculating Moments

This is the equation for calculating the size of a moment:

Where:

 moment of a force, M, in newton-metres, Nm


 force, F, in newtons, N
 distance, d, is the perpendicular distance from the pivot to the line of action of the force, in metres, m.

When we are calculating the size of a moment, it is very important to remember that the force is
always acting at right angles to the distance measured (Fig 2).
Balancing Moments

Question: What is the moment of a 4N downwards force acting 0.4m from a pivot?

1. Draw a diagram.
To tackle this question, we should start off by drawing a diagram.

Balancing Moments

2. Look at the formula.


Now that we have a visual representation, we can see that our force is perpendicular to the
distance we were given. Now we can use our formula:

M = Fd

3. Substitute in the numbers.


Once we have put the numbers into the equation, we should get our answer.

M = 4 x 0.4

M = 1.6 Nm
Changing the Size of a Moment
 Moments can be increased. We can increase the size of a moment by either increasing the force
applied, or increasing the perpendicular distance from the pivot.
 Moments can be decreased. We can decrease the size of a moment by either decreasing the force
applied, or decreasing the perpendicular distance from the pivot.

Balancing Moments
Clockwise and Anticlockwise Moments

Since moments cause objects to rotate, we can classify them by their direction. The two terms
that we use to describe the direction of a moment are clockwise and anticlockwise, as in Fig 3.

Balancing Moments

Balancing Moments

In certain situations, moments can balance each other out and the object will stay still (instead of
turning). For this to be the case, the total clockwise moment will be equal to the total
anticlockwise moment about a pivot.

If the total clockwise moment doesn’t equal the total anti-clockwise moment, the object will
rotate:
Balancing Moments

Calculating Force and Distance from Moments

Previously, we looked at calculating a moment from a force and a distance. Now, we are going to
work backwards, using the moment to find the force or distance.

In these situations, we are going to use the fact that the system is balanced. As we mentioned
before, in a balanced system, the total clockwise moment will equal the total anticlockwise
moment. By using this principle, we are able to form an equation which will help us to solve
questions:

Question: Sam and Jess are sat at opposite ends of a seesaw. Jess weighs 300N and is sat 3
metres away from the centre of the pivot. Sam weighs 450N. Whilst they are sat in their current
positions, the seesaw is balanced. How far away is Sam sat from the centre of the pivot?

1. We need to draw a diagram.


It is very important that we draw a clear diagram to see exactly what is going on. Add in all the
values for the weights and distances that we know. We will simply label the distance we are
trying to find as ‘d’.

Balancing Moments

2. Assign directions to the moments.


In this case, we can see from the diagram that the Sam is creating an anticlockwise moment,
whereas Jess is creating a clockwise moment.

3. Calculate the moment that we have information for.


In this question, we can calculate Jess’s moment. We can do this by using the equation

M = Fd
M = 300 x 3
M = 900 Nm

4. Remember that anticlockwise moments = clockwise moments.


The seesaw is balanced, so we know that the anticlockwise moments are equal to the clockwise
moments.

5. Substitute numbers into the equation.


We have already calculated Jess’s moment, so we just need to put in the missing information for
Sam.

We can use this information to help us create a balanced equation:

anticlockwise moment = clockwise moment


Sam’s moment = Jess’s moment
Fd = Fd (Since M = Fd)
450 x d = 300 x 3
450 x d = 900
d = 900 / 450
d = 2m

Therefore, we can say that Sam is sat 2m away from the centre of the pivot.
Multiple Forces
Question: (continued from previous question)… Sam moves his position on the seesaw, and
moves his bag from the ground onto his side of the seesaw. The distance between pivot and Sam
is double of the distance between pivot and the bag. The bag weighs 100N and Sam weighs 450
N. Jess has not moved – she is 300N and 3m away from the pivot on the other side. How far is
the bag from the pivot? The seesaw is still balanced.

1. We need to draw another diagram.


Draw an updated diagram. The distance from the pivot to the bag can be called d and therefore
the distance from the pivot to Sam is 2d.

Balancing Moments

2. Assign directions to the moments.


In this case, we can see from the diagram that Sam and the bag are creating an anticlockwise
moment, whereas Jess is creating a clockwise moment.

3. Calculate the moment that we have information for.


In this question, we can calculate Jess’s moment. We know this is 900 Nm from the previous
question.

4. Remember that anticlockwise moments = clockwise moments.


The seesaw is balanced, so we know that the anticlockwise moments are equal to the clockwise
moments.

5. Substitute numbers into the equation.


We have already calculated Jess’s moment. We know

anticlockwise moment = clockwise moment

Sam + Bag moment = Jess’s moment = 900


Sam’s moment = 450 x 2d = 900d
Bag’s moment = 100 x d = 100d
Sam + Bag moment = 1000d
1000d = 900
d = 0.9
Therefore, we can say that the bag is 0.9m away from the centre of the pivot, and Sam is 1.8m
away.

Objects in Equilibrium
Method
1. Gather your equipment. You will need a beam, a ruler, two newton metres, support for the beam and
an object to hang from the beam.

Balancing Moments
2. Set up the equipment. Balance the beam using the supports and attach the object to the beam.
3. Replace the supports. Once the beam is in equilibrium, replace the supports with the two newton
metres on each side of the beam.
4. Find the forces. Using the newton metres take readings for the forces acting on the beam.
5. Find the distances. Using a ruler, measure the perpendicular distance of each force acting on the beam.
6. Find the anticlockwise and clockwise moments. Using the forces and distances, find the moments.
The anticlockwise moment should equal the clockwise moment.
FAQs
→What are balancing moments in physics?
Balancing moments, also known as torque, refer to the force that causes an object to rotate around a fixed
point. In physics, balancing moments are important when considering the stability of an object and
whether it will tip over or stay upright.
→What factors affect balancing moments?
The factors that affect balancing moments include the position of the object’s center of mass, the length
of the lever arm, and the size of the applied force. A longer lever arm or a larger force will produce a
greater torque, making it harder for an object to ba

→How can you balance an object?


You can balance an object by ensuring that the weight is evenly distributed and the center of mass is
positioned over the point of support. This way, the applied forces will produce equal and opposite
torques, preventing the object from tipping over.
→What is the relationship between force and torque?
Force and torque are related in that a force can produce a torque, depending on its position relative to the
point around which an object is rotating. The greater the force, the greater the torque it will produce, and
vice versa.
→How is torque calculated in physics?
Torque can be calculated using the following formula: torque = force x lever arm. The lever arm is the
distance from the point of rotation to the point where the force is being applied.
→What are some real-life examples of balancing moments?
Some real-life examples of balancing moments include a see-saw, a door hinge, and a spinning ice skater.
In each of these examples, the forces applied produce torques that must be balanced in order to maintain
stability.
→How does the size of an object affect its balancing moments?
The size of an object does not directly affect its balancing moments, but it can affect the position of the
object’s center of mass. A larger object may have a more diffuse center of mass, making it more difficult
to balance than a smaller, more compact object.
→What role do balancing moments play in physics?
Balancing moments play a crucial role in physics, as they help us understand the stability of objects and
how forces can cause them to rotate. Understanding balancing moments is essential for studying fields
such as mechanics, engineering, and sports science.

Work Done in a Spring


Energy Transfers
 Forces do work on springs. When a force acts on a spring to compress or stretch it, the force is
doing work. We previously mentioned that doing work is simply another way to talk about the transfer
of energy.
 Energy is transferred into elastic potential energy in the spring. As the force acts on a spring, all of
its energy will be transferred into elastic potential energy (EPE). This elastic potential energy will
become stored in the spring and transferred into another form of energy later on.
 Usually the work done is equal to the EPE stored. When an object is elastically deformed, all of its
energy will be transferred into EPE. Therefore the work done will be equal to the EPE stored.
However, if the object is in-elastically deformed (i.e. we have passed the limit of proportionality), this
isn’t the case.

Calculating Work Done in a Spring

When a spring changes shape, for example when stretched or compressed, we can calculate the
amount of work done and energy transferred to EPE.

Remember, we can only calculate the work done up to the limit of proportionality (the point at
which force and extension are no longer in a proportional relationship).

This is the equation for calculating the work done when compressing or stretching a spring:

Where:

 Ee is elastic potential energy, measured in joules, J


 k is the spring constant, measured in Newtons per metre, N/m
 e is the extension, measured in metres, m.

Work Done in a
Spring
The area under the force-extension graph is equal to the potential energy stored. This is the
graph with the straight line not the curved line, because if we go beyond the limit of
proportionality we cannot calculate elastic energy (or we can, but you don’t need to know about
it!).

Question: A spring is stretched by 50cm. The spring constant for the spring is 30 N/m. What is
the elastic energy (in J) stored in the spring?

1. Convert the extension into the correct units.


The spring constant is in newton-metres, so our extension needs to be in metres.

50cm = 0.5m.

2. Calculate the elastic energy

Ee = 0.5 ke²
E = 0.5 x 30 x 0.5 = 7.5J

Question: What is the relationship between work done and elastic` potential energy in a spring
which is being stretched? (2 marks)

Mark 1 – Work done is proportional to elastic potential energy stored in the spring.

Mark 2 – Until the limit of proportionality.

Investigating Force and Extension of a Spring


Method
1. Set up a clamp stand with a spring. Attach a spring to the end of a clamp stand. Make sure that there
are no weights hanging off the end of the spring.
2. Measure the unextended spring. Place a ruler just behind the spring, so that you can read off a value
for the length of this unextended spring. Make a note of this value, ensuring that the unit you record is
metres.
3. Add a weight to the spring. Next, attach a weight to the other end of the spring. The spring will
extend.
4. Measure the extension. Wait until the spring has stopped moving and carefully read off a new value
for spring length. Make sure that you are reading off your values at eye level to prevent any errors from
occurring.
5. Record your results. Record your results in a table with the following headings:

Work
Done in a Spring
6. Repeat with several different weights. Continue to add weights and record your measurements in the
table. Stop after you have at least 7 measurements.
7. Plot a force-extension graph. Using your results, plot a graph of force against extension. Draw on a
line of best fit, which should go through the point (0,0).
8. Calculate the spring constant. You can use your force extension graph to find a value for the spring
constant. Calculating the gradient of the graph gives a value for the spring constant, k , as explained in
Fig 11.

Work Done in a
Spring
→What is Work Done in a Spring?
Work Done in a Spring is a concept in physics that refers to the energy transferred to or from a spring
when it is compressed or stretched. It is calculated by multiplying the force applied to the spring by the
distance it is moved.
→What is Hooke’s Law?
Hooke’s Law is a principle in physics that states that the force required to extend or compress a spring is
proportional to the distance it is moved. This means that the more you compress or stretch a spring, the
more force it will take to keep it compressed or stretched.
→How is Work Done calculated in a Spring?
Work Done in a Spring is calculated using the equation: Work Done = Force x Distance. The force is the
force applied to the spring, and the distance is the distance that the spring is moved.
→What is Elastic Potential Energy?
Elastic Potential Energy is the energy stored in a spring when it is compressed or stretched. When a
spring is compressed or stretched, it stores energy that can be used to perform work when it is released.
→What is the relationship between Work Done and Elastic Potential Energy?
The Work Done on a spring is equal to the change in Elastic Potential Energy of the spring. This means
that when you do work on a spring, you are transferring energy into the spring, which is stored as Elastic
Potential Energy.
→Can Work Done in a Spring be negative?
Yes, Work Done in a spring can be negative. This occurs when the force applied to the spring is in the
opposite direction to the displacement, causing the spring to release energy.
→Can the same amount of Work Done be done on a spring with different forces?
No, the same amount of Work Done cannot be done on a spring with different forces. The amount of
Work Done depends on both the force applied to the spring and the distance it is moved.
→How does the stiffness of a spring affect Work Done?
The stiffness of a spring, also known as its spring constant, determines the amount of force required to
compress or stretch the spring a certain distance. A spring with a high stiffness requires more force to
compress or stretch than a spring with a low stiffness. This means that the amount of Work Done in a
spring depends on its stiffness.

Changes in Momentum
Forces and Momentum
 Forces change the speed or direction of moving objects. When objects are moving, forces can act on
them. These forces can change the speed of an object, or change the direction in which the object is
moving. In this way, forces can change the velocity of an object.
 Forces can therefore change momentum. Since forces can change the velocity of an object, this
means they must also be able to change the momentum of an object. This is because momentum =
mass x velocity, so if the velocity changes, then the momentum also changes.

Combining Equations
For exams, you need to be able to show that force is equal to the rate of change of momentum.
We can do this in the following way.

1. We need to use two equations.


For this proof, we will be combing two equations we have seem previously.

F=m×a
a = (v-u) / t

2. Rearrange both equations.


Now that we have both equations, we need to rearrange them to get ‘a =’.
F=m×a
a=F/m
a = (v-u) / t

3. Make both equations equal.


We can now equal both the equations to each other and rearrange to get ‘F =’.

a=F/m
a = (v-u) / t
F/m = (v-u) / t
F = m(v-u) / t
Since (v-u) = ∆v
F = m∆v / ∆ t
F /∆ t = m∆v

In other words, the impulse is equal to the change in momentum. The impulse is the product of
average force and and time of a contact in a collision.

Safety Features and Momentum


In everyday life, a lot of safety features are designed based on momentum. To explain this, we
can look at the equation we just learnt:

F = m∆v / ∆ t

We can decrease the force on an object by increasing the time taken for the change in momentum
to happen, as demonstrated below:

F = m∆v / ∆ t

Example 1. When the momentum is 30 kg m/s and the time taken is 3 seconds:

F = 30 / 3
F = 10 N

Example 2. When the momentum is 30 kg m/s and the time taken is 6 seconds:

F = 30 / 6
F=5N

As you can see, when the time increases from 3 seconds to 6 seconds, the force experienced by
the object will decreases significantly. In other words, as the rate of change of momentum
decreases, the force experienced by the object will also decrease.
The following safety measures use the principle of decreasing the rate of change of momentum
in order to decrease the force experienced by objects:

1. Air Bags

Air bags will inflate in the event of a car crash. The passenger will naturally move forwards due to
impact, but instead of hitting the hard dashboard, they will hit the air bag. The air bag will absorb some
of the impact, slowing down the passenger considerably. By increasing the time taken for the
passenger to move, the air bag will decrease the rate of change of momentum, therefore decreasing the
force on the passenger’s body.
2. Seat Belts

Seat belts are worn as a safety measure by all passengers in a car. These are often stretchy, allowing for
the passenger to move slightly before coming to a stop. In this way, seat belts will increasing the time
taken to come to a stop, and hence decrease the rate of change of momentum and therefore decrease
the force experienced by the passenger.
3. Gymnasium Crash Mats

Crash mats are commonly used by gymnasts to reduce the force of impact. Since these mats are
compressible, a gymnast will ‘sink’ into the mat slightly and take a longer time to come to a complete
stop. Therefore they increase the time taken to come to a stop, and therefore decrease the rate of
change of momentum and therefore a decrease in the force experienced when the gymnast lands on the
floor.
4. Cycle Helmets

Inside a cycle helmet, there is usually a layer of foam. This foam acts very much in the same way as
crash mats; it is a compressible layer which reduces the force of impact by increasing the time
taken to come to a stop. By doing so, we can reduce the chances of any head injuries.

5. Cushioned Surfaces

Many playgrounds often have slightly softer, cushioned surfaces. They are compressible, so if a child
falls over, they will take longer to completely stop moving. The surface increases the time taken to
come to a stop. This reduces the chance of the child injuring themselves.

Linking force, mass, velocity and acceleration


In this chapter, we have seen various relationships to do with force, mass, velocity and
acceleration.

We have already linked force, mass and velocity through the following relationship:

F = m∆v / ∆ t

We can now link acceleration to this relationship, to gain an equation we have seen before:

Since a = ∆v / t
So F = ma
As you can see, this means that the equations to do with force, mass, velocity and acceleration
are linked. In exams you may be given questions where you are expected to use more than one
equation, or rearrange equations to find a missing value.

Forces and Elasticity


Forces Acting On Objects
Forces can cause stretching, bending or compression of an object. You need to understand each
of these three effects, and give examples of forces causing each.

Stretching, bending and compression require multiple forces to be acting on an object. For
example, if you just pushed a spring, then it would move in a certain direction. But if you pushed
both ends of the spring, the spring would compress.

Forces and Elasticity

Stretching an Object
 Stretching is caused by pulling forces. To stretch a spring, we have to pull on it. This can be done by
adding some small weights to the end of the spring, making it change in length.
 Pulling causes a spring to stretch. As the weights are added onto the spring, the length of the spring
will increase. The spring has stretched from its original length to a new, longer length. This is
called extension.

Compressing an Object
 Compression is caused by pushing forces. To compress a spring, we have to push it. This can be done
by pushing the two ends of the spring together, making it change in length.
 Pushing causes a spring to squash. As the two ends are pushed together, the length of the spring will
decrease. The spring has been compressed from its original length to a new, shorter length. This is
called compression.

Bending an Object
 Bending forces can be push or pull. A force that causes a bend can either be a push force (Fig 2) or a
pull force (Fig 3).
 The centre of an object will bend the most. When a force causes an object to bend, the centre of the
object will be the ‘bendiest’ part. This causes the ends of the object to move closer towards each other.
This is shown in Fig 16; the centre of the object is bending, whilst two ends move towards each other
(in this case, they are moving upwards).

Forces and Elasticity

Forces and Elasticity

Changing the Shape of Stationary Objects


Applying One vs., Multiple Forces

For exams, we need to learn the effects of different forces on stationary objects. We discuss
above that stretching, bending and compressing require multiple forces.

 Applying a single force causes movement. When a single force is applied to a stationary object, the
object will move in the direction of the force. There is no change of shape, since only one force has
been applied.
 Applying multiple forces causes change of shape. When multiple (two or more) forces are applied to

a stationary object, r
Forces and Elasticity
 Changes of shape include stretching, compressing and bending. We previously discussed examples
of stretching, compressing and bending in the section ‘Forces Acting On Objects’. Now, we know that
all of these changes require two or more forces to be applied.

Elastic and Inelastic Deformation


 Deformation is a change in shape. An object is said to be deformed when it has changed shape and
length, due to the effect of forces. There are two types of deformation: elastic and inelastic.
 Elastic deformation is reversible. If an object completely returns to ‘normal’ once all forces are
removed, it is said to be elastically deformed. By normal, we mean that the object has completely
returned to its original shape, size and length.
 Inelastic deformation is irreversible. if an object doesn’t completely return to ‘normal’ once all forces
are removed, it is said to be in elastically deformed. Another term used to describe inelastic
deformation is plastic deformation.
Forces and Elasticity

The Relationship between Force and Extension


Hooke’s Law

Hooke’s law is used to describe the relationship between force applied to an elastic object and
the extension of the elastic object. It is a linear relationship, where force is directly proportional
to the extension. If an object doesn’t obey Hooke’s law, there is a non-linear relationship
between force and extension.

Where:

 Force, F, in newtons (N)


 spring constant, k, in newtons per metre (N/m)
 extension, e, in metres (m)

Extension is the difference between the original length and the new, longer length.
We will learn about the spring constant below.

The Spring Constant


Linear Cases

The spring constant is used to describe the ‘stiffness’ of a spring. If the spring constant is very
high, it means that the spring is very stiff.

Forces and Elasticity

For exams, we need to calculate the spring constant in linear cases.

The spring constant can be found by rearranging Hooke’s Law.

Hooke’s Law:

We want to find k, so we can rearrange the equation to give:

Calculating The Spring Constant


Question: A force of 10N is applied to a spring that is 50cm long. The spring extends to a value
of 70cm. Calculate the spring constant, with units.

1. Write down the equation for Hooke’s Law.

F=ke

2. Find the extension.


The units of extension are in metres, so we will have to convert from cm to m in this example.

20cm = 0.2m and 70cm = 0.7m

The extension is 0.7 – 0.2 = 0.5m

3. Rearrange the formula.

F/e=k

4. Substitute for numbers.


Using the information given in the question, put in the value for force. The value for extension is
the number we have just calculated.

5. Give the correct units.


In this question, we are asked to find the spring constant with its units.

Spring Constant = 20 N/m

Relationship Between Force and Extension


Direct Proportionality
 Extension occurs due to force. When we apply force to an elastic object (such as a spring) it
will extend. The extension of an elastic object can be described by Hooke’s law.
 Force and extension are directly proportional. When we increase the force, the extension will also
increase by the same amount. This is called direct proportionality, which is often obeyed by elastic
objects such as springs.
 Direct proportionality forms a straight line graph. As a graph is plotted of force (on the y axis)
against extension (on the x axis), a straight line will start to form. This straight line will go through the
origin, since when there is no force applied, there will be no extension.
Forces and Elasticity

Limit of Proportionality

The limit of proportionality is when the spring stops obeying Hooke’s Law. It is the point
where the force applied to the spring is too large for the spring to handle. As a result, the
extension of the spring increases drastically, and the two variables are no longer in a proportional
relationship.

We learnt before about elastic and non-elastic deformation. After the limit of proportionality, the
spring has non elastic deformation, and will not return to its original length once the load is
removed.
Forces and Elasticity

For exams, there are several important things to remember about the limit of proportionality:

Forces and Elasticity


Forces and Elasticity

You need to be able to describe the difference between a linear and non-linear relationship
between force and extension:

 Linear relationships obey Hooke’s Law. Previously, we mentioned that there is a linear relationship
between the force and extension of elastic objects. This linear relationship can be described by Hooke’s
Law.
 Non linear relationships do not obey Hooke’s Law. Past the limit of proportionality, elastic objects
will not obey Hooke’s law. This means that there is no longer a non-linear relationship between force
and extension.

The Relationship Between Force and Compression


Compression vs. Extension
 Compression and extension are opposites. We know that extending a spring means increasing its
length, so compressing a spring will decrease its length. The amount of force applied will be the same
in both scenarios, but the direction of the force will be different (Fig 9).
 Extension is the difference between the original length and the new, longer length.
 Compression is the difference between the original length and the new, shorter length.
Forces and Elasticity

Compressing Elastic Objects


Similar to extension, we can use Hooke’s law to describe the compression of an elastic object.
This means that there is a directly proportional relationship between the force on an object and
the compression it experiences.

The equation linking force and compression is:

Where:

 Force, F, in newtons (N)


 spring constant, k, in newtons per metre (N/m)
 compression, c, in metres (m)

→What are Forces?


Forces are pushes or pulls that can change an object’s motion or shape. Forces can be caused by many
things, including friction, gravity, and magnetic attraction.
→What is Elasticity?
Elasticity is the property of an object that allows it to return to its original shape after it has been
deformed by an applied force. Elasticity is related to the elastic limit, which is the maximum amount of
force that can be applied to an object before it permanently deforms.
→What is Elastic Potential Energy?
Elastic potential energy is the energy stored in an object when it is stretched or compressed. This energy
is stored as a result of the elastic forces within the object, and it can be calculated by multiplying the force
applied to the object by the distance it is stretched or compressed.
→What is Hooke’s Law?
Hooke’s Law is a scientific principle that states that the extension of a spring is directly proportional to
the force applied to it, as long as the spring remains within its elastic limit. This law is named after Robert
Hooke, who first described it in the 17th century.
→What is the Elastic Limit?
The elastic limit is the maximum amount of force that can be applied to an object before it permanently
deforms. When an object is subjected to forces beyond its elastic limit, it will not return to its original
shape after the force is removed.
→How does Elasticity relate to Forces?
Elasticity and forces are closely related because forces can cause objects to change shape, and the ability
of an object to return to its original shape after being deformed by a force is related to its elasticity. When
a force is applied to an object, it will deform and store elastic potential energy, and when the force is
removed, the object will return to its original shape as the stored energy is released.
→Why is the study of Forces and Elasticity important in GCSE Physics?
The study of forces and elasticity is important in GCSE Physics because it is a fundamental concept that
is required to understand many other aspects of physics, such as mechanics and waves. Understanding
how forces affect the motion and shape of objects, and how elasticity can be used to describe and predict
the behavior of objects under stress, is essential for success in GCSE Physics.

Momentum
Momentum
Calculating Momentum

The momentum of an object is a measure of how hard it is to stop. It depends on two factors:

1. Mass – the heavier an object, the harder it is to stop.


2. Velocity – the faster an object is moving, the harder it is to stop.

We can calculate momentum using the following equation:


Where:

 momentum, p, in kilograms metre per second, kg m/s


 mass, m, in kilograms, kg
 velocity, v, in metres per second, m/s

Question: Calculate the momentum of an 8kg object travelling at a speed of 2 m/s.

1. Write out the equation.

p = mv

2. Substitute in the numbers.

p=8x2
p = 16 kg m/s

Conservation of Momentum
Conservation of Momentum
Momentum is a very important concept in Physics, and it is a property of all moving objects.

There is a key principle that you should know in relation to this topic – the conservation of
momentum:

In a closed system, the total momentum before an event is equal to the total momentum
after the event.

Closed and Open Systems

This principle is for a closed system. Remember, a closed system is one which does not
exchange energy with the surroundings.
In real life, if there was a car crash, for example, it would be an open system so lots of energy
would be lost as heat and sound energy to the surroundings. However, in exams we will normally
be assuming that it is a closed system to make calculations easier.

Conservation of
Momentum

Examples of Momentum
Qualitative Scenarios of Momentum

Below we will practice some examples of momentum calculations.

Example Scenario: Imagine two skaters on an ice rink. Skater A is stationary, whilst Skater B is
moving with a velocity. Skater B collides into Skater A, causing both skaters to move off
together with a lower velocity. Momentum is conserved.

In this scenario, momentum has been conserved.

Momentum Before Collision

Before the two skaters collided, the momentum is: Mass of Skater B x Velocity. We do not take
into account A, because it cannot have a momentum as it has a velocity of 0.
Conservation of Momentum

Momentum After Collision

After the collision, the momentum is: (Mass of A + B) x Velocity. The mass of the system will
have increased because it is now (A + B) rather than just B. If momentum is to stay the same, the
velocity must decrease.

Conservation of Momentum

Quantitative Scenarios of Momentum

In exams, you may be asked to perform calculations involving momentum. We can elaborate on
the previous example to demonstrate this.

Question: Imagine two skaters on an ice rink. Skater A (mass 45 kg) is stationary, whilst Skater
B (mass of 60kg) is moving with a velocity of 6 m/s. Skater B collides into Skater A, causing
both skaters to move off together with a lower velocity. Assuming that momentum is conserved,
what is the new lower velocity, to 1 d.p.?

1. Draw a diagram.

Conservation of
Momentum

2. Calculate initial momentum.


This is the momentum before the collision occurs.

Initial momentum = mass x velocity


Initial momentum = 60 x 6
Initial momentum = 360 kg m/s

3. Calculate the final momentum.


We do not know the velocity, so we can substitute this with the symbol ‘v’.

Final momentum = mass x velocity


We need to calculate the total mass of both skaters when they move off
together.
This is 45 + 60 = 105 kg.
Final momentum = 105 x v

4. Find ‘v’.
By using the concept of conservation of momentum, we know that the initial momentum = final
momentum. We can use this to find ‘v’.
Initial momentum = Final momentum
360 = 105 x v
v = 360 / 105
v = 3.428571
v = 3.4 m/s (1dp)

Factors Affecting Braking Distance


Adverse Conditions
 Adverse conditions can have a large effect on the stopping distance of a
vehicle. Usually, they will lead to an increase in both the thinking and braking
distances. Overall, these two will combine, resulting in a larger overall stopping
distance in adverse conditions.
 Adverse conditions include wet or icy conditions, which will increase the
stopping distance of a vehicle. The ice and rain on the road can lead to the car
skidding, or ‘aquaplaning’. This can be very dangerous because the driver may
lose control of the car, resulting in a car crash.

Vehicle Conditions

It is very important for the driver to keep their vehicle safe and functioning. Two things of
particular importance are:

 Condition of Brakes – the driver of a car must keep their brakes in good
condition. If the brakes aren’t in good condition, then the vehicle won’t stop
quickly enough in an emergency.
 Condition of Tires – the driver of a car must keep their tyres in good condition.
If the tyres aren’t in good condition, they can contribute to skidding in adverse
weather conditions.

Emergency Stops and Safety


 Whether a driver is stopping in an emergency or not, they need to be able to
stop safely. This means that they do not cause harm to themselves or others.
 We know that the faster a vehicle is travelling, the longer distance it takes to
stop. This can be affected by a variety of factors, some of which we’ve already
explored. These include alcohol, weather conditions and tiredness.
 The stopping distance of a vehicle can affect the safety of the driver. If a
vehicle has a very large stopping distance, then the driver may have a difficult
time stopping the vehicle before an accident occurs. This is why we have speed
limits, so that drivers can stop safely when necessary.

Vehicle Stopping Distances


Different vehicles have different stopping distances. This is due to the size of the vehicle, the
weight of the vehicle and even the grip on the tyres. For exams, you will need to know the
stopping distances for vehicles over a variety of speeds.

Factors Affecting
Braking Distance

How Brakes Work


In order to stop a vehicle from moving, a force must be applied to the brakes. This force is
created by the driver pressing down on the brakes. As we have previously discussed, forces can
result in work being done. This means that when the driver presses on the brakes, they are doing
work. This has two effects:

 Reduces kinetic energy of tyres – as work is being done, the kinetic energy of
the tyres will be reduced. As the kinetic energy decreases, the tyres will move less
quickly. The kinetic energy from the tyres will be transferred into other forms of
energy in the brakes.
 Increases temperature of brakes – some of the kinetic energy from the tyres
will be transferred into thermal energy in the brakes. As the thermal energy of
the brakes increases further, the brakes will get hotter. This means that the
temperature of the brakes has increased.

Factors Affecting
Braking Distance

Braking at High Speeds


Difficulty of Breaking at High Speed

 Speed makes braking difficult. When travelling at high speeds, it is difficult to


brake over a small distance. We know that the higher the speed, the longer it takes
for a vehicle to brake.
 Higher speeds require larger braking forces. When travelling at high speeds,
vehicles will require a larger braking force to slow them down. Large forces will
result in more work being done, so slowing down the car more quickly.

Factors Affecting
Braking Distance

Large Decelerations
 Large braking forces lead to large decelerations. When a large braking force is
used, the vehicle will slow down very rapidly. ’Slowing down’ is known
as deceleration, and large braking forces will lead to large decelerations.
 Large decelerations affect the brakes. When a vehicle has a large deceleration,
there is a significant effect on the brakes. In order to slow down the vehicle, a
very large braking force is being applied, which will lead to lots of work being
done. In turn, this work done will lead to lots of thermal energy in the brakes,
potentially leading to them overheating.

Forces and Deceleration


In exams, you may be asked about the forces involved in a vehicle’s deceleration.

During deceleration, we know that the resistive forces will be greater than the driving force of
the vehicle.

Therefore there is a resultant force going backwards. The car is still moving forwards, but is
decelerating (remember resultant forces cause a change in speed).

You may be asked to estimate the size of these forces in an exam, such as the following example.

Question: A car travelling at average speed has to brake suddenly to avoid a child running into
the road 15 metres away. Estimate the braking force involved in the car’s deceleration.

1. We know the average speed.


We know that the average speed of a car is about 30 m/s, from our table earlier on in this chapter.

Speed of car = ~ 30 m/s

2. We are assuming uniform deceleration.


Given this scenario, we need to assume that the deceleration is uniform to perform any kind of
calculation.

3. We need to find the deceleration.


Now that we have a speed for the car, we can use our equation v² – u² = 2as. We need to
rearrange this to find the deceleration.

v² – u² = 2as.
(v² – u²) / 2s = a

4. Substitute in the numbers.


(v² – u²)/ 2s = a
(30² – 0²) / 2 x 15 = a
900 / 30 = 30
a = 30 m/s²

5. Estimate the mass of the car.


Now that we have the deceleration of the car, we need to estimate the mass of the car.

Mass of a car = ~ 1200 kg

6. Use F = ma.
We have a mass and a deceleration, so now we can substitute in the numbers.

F = ma
F = 1200 x 30
Force = ~ 36 000 Newtons

FAQs
→What is braking distance in physics?
Braking distance is the distance a vehicle travels before it comes to a complete stop after the brakes are
applied.
→What are the factors that affect braking distance?
The following are some of the factors that affect braking distance: speed of the vehicle, road conditions,
tire condition, weight of the vehicle, and the efficiency of the braking system.
→How does the speed of the vehicle affect braking distance?
The higher the speed of the vehicle, the longer the braking distance will be. This is because a faster
moving vehicle requires more time and distance to come to a stop.
→How do road conditions affect braking distance?
Road conditions such as rain, snow, ice, and gravel can increase the braking distance. These conditions
can cause the tires to lose traction, making it harder for the vehicle to come to a stop.
→How does tire condition affect braking distance?
Worn tires can decrease the traction of the vehicle, making it harder to come to a stop. Properly inflated
tires with adequate tread depth will improve braking performance and decrease braking distance.
→How does the weight of the vehicle affect braking distance?
The heavier the vehicle, the longer the braking distance will be. This is because more force is required to
stop a heavier vehicle.
→How does the efficiency of the braking system affect braking distance?
The better the efficiency of the braking system, the shorter the braking distance will be. Modern vehicles
have improved braking systems, such as anti-lock brakes, that can help reduce braking distance.

These questions and answers can help students understand the factors that affect braking distance and
prepare them for their GCSE physics exams.
Newton’s Third Law
Newton’s Third Law
The third law you need to know is Newton’s Third Law.

Equal and Opposite Forces


 Forces can be exerted on objects. These forces can be exerted by objects onto each other, or by other
surfaces. The forces can make objects move or keep them in place.
 Newton’s Third Law states that when two objects interact, the forces they exert on each other are
equal and opposite. This means that for every action, there is an equal and opposite reaction.

Example: Car Crash

An example of this is a car crash.

Car A will exert a force onto Car B, whilst Car B exerts an equal and opposite force on Car A.
For example, the yellow car exerts the yellow force on the red care

Newton’s Third Law

The two cars may accelerate away from each other due to the impact:
Newton’s Third Law

Equilibrium Situations
 Equilibrium situations result in no movement. When an object is in equilibrium, it will not move as
the forces are balanced. When one force acts on an object, it will be balanced out by an equal and
opposite force.
 Pushing against a surface is equilibrium. When a person pushes against a surface, such as a desk,
they will experience Newton’s Third Law. The person exerts a force on the desk, whilst the desk exerts
and equal and opposite force on the person.

Reaction Time
Normal Reaction Times
As previously discussed, different people will have different reaction times. A typical reaction
time could be anywhere from 0.2 – 0.9 seconds. In this section, we will explore factors affecting
reaction times in more detail.
Factors Affecting Reaction Times

The following factors can all affect the ability of a driver to react to hazards on the road.

1. Tiredness – the more tired you are, the more difficult it is for you to react. This increases reaction
times, therefore leading to an increase in stopping distance.
2. Drugs – drugs can affect your ability to react when driving. For example, taking some medications can
make you feel drowsy, resulting in an increased reaction time.
3. Alcohol – drinking alcohol can impair a driver’s ability to react. It increases reaction times, leading to
an increase in stopping distance.
4. Distractions – distractions, such as talking on the phone, can lead to a driver losing concentration on
the road. This leads to an increase in reaction times and once again, an increase in stopping distance.

Evaluating Effects of These Factors

In exams, you may be given some data relating to the factors that we have just discussed. Using
this data, you should be able to compare these and evaluate each of their effects.

Measuring Reaction Times


We can measure reaction times in the following way.

1. Gather the equipment. You will need a metre ruler.


2. Choose a subject. Ask someone to sit at a desk, with their hand out.
3. Line up the ruler. You should hold the ruler up, but make sure that the ruler is not touching the
person’s hand. The ‘zero’ of the ruler should be in line with the person’s thumb and forefinger, as if
they are getting ready to catch it.
4. Drop the ruler. Without telling the person, drop the ruler.
5. Measure the distance. At the point where the person catches the ruler, read off the distance. This will
give you a distance.
6. Use acceleration due to gravity. We know that the acceleration due to gravity is 9.8 m/s2. Using this
value, we can find the value for time, since we know the equation ‘acceleration = change in velocity /
time taken’. However, firstly we need to find the change in velocity.
7. Find the change in velocity. We can use an equation that we learnt previously to find the change in
velocity : v2 − u2 = 2 a s. Rearranging this equation, we can make v the subject of the formula and
therefore we can find the final velocity. Knowing the initial velocity, we can find the change in
velocity.
8. Find the time. Using the change in velocity and acceleration, we can find the time using the following
equation: a = ∆v / t. We can rearrange the formula to make ’t’ the formula of the subject, which is the
reaction time of the person.
9. Compare to typical results. Typically, human reaction. times are anywhere between 0.2 and 0.9
seconds. You need to be able to remember these values for your exams.
Interpreting Measurements
 Measurements may not be accurate. When performing this experiment, results may not always be
entirely accurate. In order to improve the accuracy, you should carry out plenty of repeat readings.
This will help you to remove any anomalies that you come across.
 People may be distracted. If the experiment room is noisy, or the person in the experiment is not
feeling well, they may be distracted. This could have an impact on the reaction times and often
increase them to larger than expected values.
 Measurements must be made precisely. Since we are using a ruler to take measurements, we must
make sure that they are precise. The divisions on the ruler must be at least in millimetres (the smaller
the measuring unit, the more precise the results).

→What is reaction time?


Reaction time is the amount of time it takes for a person to respond to a stimulus. It is often measured in
seconds or milliseconds.
→Why is reaction time important?
Reaction time is important because it can impact a person’s ability to perform daily tasks, such as driving,
playing sports, or operating machinery. A slow reaction time can increase the risk of accidents or
mistakes.
→How is reaction time measured?
Reaction time is usually measured using a reaction time test, where a person is asked to respond as
quickly as possible to a visual or auditory stimulus. The time taken for the person to respond is then
recorded and used as a measure of their reaction time.
→What factors affect reaction time?
There are several factors that can affect reaction time, including age, fatigue, alcohol and drug use,
illness, and attention level. Physical factors, such as reaction distance and response method, can also
impact reaction time.
→How can reaction time be improved?
Reaction time can be improved through regular exercise, good sleep habits, and avoiding alcohol and
drugs. Improving reaction time can also be achieved through training and practice, such as playing fast-
paced video games or performing quick response drills.
→What is the average reaction time for a person?
The average reaction time for a person is around 0.25 seconds. However, this can vary based on factors
such as age, gender, and physical condition.
→What is the difference between simple reaction time and choice reaction time?
Simple reaction time is the amount of time it takes for a person to respond to a single, straightforward
stimulus. Choice reaction time, on the other hand, is the amount of time it takes for a person to make a
decision and respond to a more complex or multiple-choice stimulus. Choice reaction time is typically
slower than simple reaction time.

Stopping Distance
Calculating Stopping Distance
The stopping distance of a vehicle is the distance it takes for a moving vehicle to come to a
complete stop, which can be calculated by adding together the thinking and braking distances:

Thinking Distance

The thinking distance is the distance travelled in the thinking time it takes to think and react to
a hazard on the road. It is the time between the driver seeing the hazard and them hitting the
breaks.

Thinking distance can be affected by several factors:

 Human variation – for humans, normal reaction time can be anywhere between 0.2 to 0.9 seconds.
This is the time it takes for a driver to react to a stimulus on the road and actually press the brakes.
From person to person, reaction times can vary drastically.
 External influences – Thinking time can also be affected heavily by alcohol, drugs, distractions and
tiredness. This is one of the big risks of ‘drink driving’ – having slow reactions.

Braking Distance

The braking distance is the distance travelled between the moment the driver hits the breaks to
the moment the car.

When a driver sees a hazard, such as a child running into the road, they may stop suddenly. This
is called an emergency stop. Once the driver presses on the brakes, it still takes the vehicle some
time to stop. This is the braking distance.

Braking distance can be affected by several factors including:

 Force of breaking – the force of braking will affect the distance it takes for the car to brake. Normally
most drivers would push down fully on the breaks, so this is less important for an emergency stop
 Speed of the vehicle – the faster a vehicle is travelling, the greater the braking force needed to stop the
car. This means that the braking distance will also increase, meaning that the total stopping distance
will also increase.
 Mass of the vehicle – if the vehicle has a very low mass, then it will travel further along a surface,
resulting in a larger braking distance
 Road conditions – if the road surface is smooth (for example when it is icy or rainy) then the vehicle
will travel further along the surface, resulting in a larger braking distance.

Example Scenarios

Question: This diagram shows the thinking and braking distance of a car driver when he spots a
junction coming up and breaks at full power. What would happen to thinking and braking
distance if a) the driver was under the influence of alcohol, b) the driver was travelling at a faster
speed.

a) Alcohol: If the driver was under the influence of alcohol, his reactions would be slower, so
thinking distance is higher. We assume speed is constant and the car is working normal, so
braking distance is unchanged.

b) Higher Speed: If the car was travelling faster, it would take longer for it to stop once the
brakes are pressed. This means that braking distance rises. In addition, thinking distance will
increase. Even though the driver’s reactions may not be compromised and his thinking time
could stay the same, he will have travelled more in that same thinking time due to his high speed.

Braking Distance
As an object (such as a car) brakes and comes to a halt, work is done to remove the kinetic
energy. This means that the work done equals the kinetic energy. Therefore we can form an
equation to calculate the braking distance.

Forming the Equation


1. Write the equation to calculate kinetic energy.

Ek=(mv²)/2

2. Write the equation to calculate work done.

W=fxd

3. Kinetic energy equals work done.

Ek=W

5. Substitute in your two equations.

(mv²)/2=fxd

Calculating the Braking Distance

Where:

 mass, m, in kilograms, kg
 speed, v, in metres per second, m/s
 force, f, in Newtons, N
 distance, d, in metres, m

Example

Question: A car has a mass of 850kg and moves at a speed of 10m/s. What will the braking
distance of the car be, if it has a breaking force of 1800N? Give your answer to 3sf.

1. Write the equation to calculate the braking distance.

( m v ²) / 2 = f x d
2. Substitute in the numbers.

(mv²)/2=fxd

( 850 x 10² ) / 2 = 1800 x d

3. Find the braking distance.


Solve for d to find the braking distance.

( 850 x 100 ) / 2 = 1800 x d


85000 / 2 = 1800 x d
42500 = 1800 x d
d = 42500/1800
d = 23.6111…
d = 23.6m (3sf)

Graphs for Stopping Distance


 The thinking distance will increase linearly as the speed increases.
 The braking distance will increase non-linearly as the speed increases. The faster a vehicle is
travelling, the longer the braking distance will be.

The main thing that you need to remember is that as the speed of a vehicle increases, the total
stopping distance will increase.

Question: A traffic analyst is scrutinising some data on three crashes that have occurred in his
country district. He is looking at 3 independent crashes here, and looking at a single vehicle from
each crash. He needs to make an early prediction on whether i) any of the drivers were under the
influence of alcohol, ii) the road conditions affected the stopping distance. He has asked you for
your opinion. Explain your observations from the graph below.
Stopping Distance

This graph is quite kind to us, as it breaks down the different stages quite nicely. You can easily
spot the thinking distance and braking distance. Let us consider each one

 The thinking distances are all quite similar for the cars. They all rise a similar amount. You might
be tempted to speculate that the lorry driver was under the influence of alcohol, and further
investigation would be useful, but realistically the thinking distances are quite similar between all 3
vehicles. Remember, there is intra-person variation which happens due to differences in reactions
from person to person.
 The braking distances are quite different for the cars. You might be tempted to speculate that the
lorry was driving on an icy road, which could explain an increased braking distance. However, you
have to remember that each vehicle has a different mass. The lorry must be very heavy, and this
explains its larger braking distance.

Overall, the results are what you would expect, so it is unlikely that alcohol or poor road
conditions were influencing factors in these crashes.

→What is stopping distance in physics?


Stopping distance is the total distance that an object travels from the time when its brakes are applied to
the time when it comes to a complete stop. It is a measure of the time and distance required for an object
in motion to stop.
→How do you calculate stopping distance in physics?
To calculate stopping distance in physics, you need to know the speed of an object, the reaction time of
the driver, and the braking distance of the vehicle. You can use the formula stopping distance = reaction
distance + braking distance to calculate the stopping distance.
→What is reaction distance in physics?
Reaction distance is the distance that an object travels from the time when a driver first sees a danger to
the time when they apply the brakes. It is a measure of the driver’s reaction time.
→What is braking distance in physics?
Braking distance is the distance that an object travels from the time when the brakes are applied to the
time when it comes to a complete stop. It is a measure of the effectiveness of the brakes and the friction
between the tires and the road.
→How does speed affect stopping distance in physics?
Speed has a major impact on stopping distance in physics. The faster an object is moving, the greater its
stopping distance will be. This is because the braking distance of a vehicle increases as its speed
increases, and so does the reaction distance of the driver.
→What are some real-world applications of stopping distance in physics?
Stopping distance is an important concept in physics that has many real-world applications. It is used in
transportation to design and maintain vehicles and roadways, to evaluate the safety of vehicles and roads,
and to develop traffic control systems. It is also used in sports to measure performance and in many other
areas where the motion of objects needs to be measured and analyzed.
→Can you reduce stopping distance by driving slower?
Yes, you can reduce stopping distance by driving slower. This is because the braking distance of a vehicle
decreases as its speed decreases, and so does the reaction distance of the driver. Driving slower can also
help you to react more quickly and effectively to road hazards, making it safer for you and other drivers
on the road.
Radiation and Temperature (GCSE Physics)
Radiation and Temperature
Emission of Radiation
 All objects emit radiation. As we discussed previously, all objects will emit radiation, not just black
bodies. However, the temperature of an object can hugely determine the amount of radiation that an
object releases.

Emission and Temperature


 Temperature can affect emission. The higher the temperature of an object, the more radiation it will
emit in a certain time. This can also affect the intensity and the wavelength of radiation emitted, as we
will see next.
 Intensity and wavelength can be affected. As the temperature increases, the intensity of the emission
will also increase. This is a directly proportional relationship. However, as the temperature increases,
the wavelength of the radiation will decrease (i.e. get shorter). This means that the temperature and
wavelength will hare an indirectly proportional relationship.
Radiation and
Temperature

Increasing Temperature
 Radiation is balanced at a constant temperature. When an object is kept at a constant temperature,
the amount of radiation being absorbed will be equal to the amount of radiation being emitted. This is
known as a balance.
 When radiation is unbalanced, object temperature rises. However, there may be some cases where
the radiation is not balanced. For example, when the temperature of an object rises, this means that the
object must be absorbing more radiation than it is emitting.
Radiation and Temperature

Temperature of Objects
 Temperature is related to radiation. We now know that the temperature of an object
is positively related to radiation. As the temperature of an object increases, the more radiation it will
emit.
 The temperature of the Earth is affected by several factors. Since the temperature of the Earth is
also controlled by the balance of radiation, this will also be affected by several factors. We will explore
these in the next section.

Perfect Black Bodies


Infrared Radiation and Temperature
 Infrared radiation always occurs. No matter what the temperature of an object is, it will always emit
and absorb infrared radiation. All objects will obey this rule.
 Heat can increase infrared radiation. If the temperature of an object is increased, the the amount of
infrared radiation it emits will also increase. More simply we can say that the hotter an object is, the
more infrared radiation it will radiate.
A Perfect Black Body
For exams, you need to know the definition of a perfect black body:

A perfect black body is an object that absorbs all of the radiation incident on it.

Defining Black Body Radiation


 Black bodies cannot reflect or transmit. Black bodies cannot reflect or transmit the wavelengths of
light. This means that black bodies will not appear transparent and they will not appear a certain colour.
Instead, all of the wavelengths are absorbed, resulting in the object being a ‘black body’.
 Black bodies are good radiation emitters. Since a good absorber is also a good emitter, black bodies
are very good at emitting radiation. In fact, black bodies are the best emitters compared to any other
objects.

Charging Materials
Charging Materials
 Certain insulating materials can become charged. You can charge certain insulating materials
by rubbing them together. For example, if you rub together an uncharged plastic rod with an
uncharged cloth, both will get charged.
 Electrons can move from rod to cloth. Electrons are negatively charged particles, which move from
the rod to the cloth during rubbing. Protons, which are positively charged particles, never move. The
rod becomes positively charged as it loses electrons. The cloth becomes negatively charged as it gains
electrons.
Charging Materials

 The positive charge equals the negative charge. The positive charge of the rod should be equal to the
negative charge of the cloth. We need to consider the number of electrons moving from the rod to the
cloth. The number of electrons gained by the cloth will be equal to the number of electrons lost from
the rod.

Static Electricity
Static Electricity
 Rubbing surfaces leads to static electricity. When we rub two surfaces together, we produce static
electricity. This occurs when electrons move from one surface to the other. We get a build up of static
positive charge on one surface, and an equal static negative charge on the other surface.
 A charge difference is created. As charge builds up on a surface, we start to create a difference in
charge between the charged surface and the earth. Usually, the earth is at zero volts. As we charge the
surface, the difference in charge between the surface and the earth will get bigger and bigger.
 Static electricity leads to sparking. Eventually, the charge difference between the earth and charged
surface gets so big that electrons will jump from the charged surface to the earth. As the electrons jump,
we will be able to see a spark.

An Example of Static Electricity


Question: Laura takes an uncharged balloon and a jumper. She rubs the balloon against the
jumper for several seconds. Laura then touches the jumper and experiences a small electrical
shock. Why has this happened?
1. When the balloon is rubbed against the jumper, we get electron transfer. As the balloon rubs
against the jumper, electrons will be transferred from the balloon to the jumper. This results in a
negatively charged jumper and a positively charged balloon.
2. When Laura touches the jumper, the electrons are transferred to her. Compared to the jumper,
Laura is positively charged. Therefore the electrons jump across the gap between the jumper and
Laura’s hand, creating a spark (and therefore an electrical shock).
3. The electrons are trying to get to the earth. Electrons are always trying to get to the earth. This
means that they will go through Laura’s body and travel straight down into the earth. In this situation,
we would refer to Laura as an ‘earthed’ object; she is in contact with the earth, therefore providing a
pathway for the electrons.

General Uses and Dangers of Static Electricity


Uses and Dangers
Static Electricity

Lightning
 Warmer air is less dense than colder air. Warm air in clouds rises, as it is less dense than colder air.
The warmer air carries small ice crystals as it moves upwards.
 An electric charge is created. As these crystals move upwards, they rub together causing an electric
charge to form at the top and bottom of the cloud.
 Lightning is caused by this electric charge. Eventually the charge becomes so large that electrons
jump to the ground, which we see us a bolt of lightning.
Static Electricity

Question:

1. The only way electrostatic charges can be produced is by the movement of electrons.
2. Static electricity causes hairs to attract each other and therefore stand up on end
3. If you rub a plastic rod with a cloth, the rod becomes positively charged, and cloth negatively charged.
4. You can charge two insulators by using friction.

Which of the above statements are true?

1. True – this is true, electrons can move but protons cannot.


2. False – this is incorrect, as electrons are transferred from the object rubbed to the hair, causing an
accumulation of charge. As the hair strands are all the same charge, negatively charged, the hair strands
repel each other and stands on end.
3. True – During rubbing, electrons move from the rod to the cloth. Protons never move. The rod becomes
positively charged as it loses electrons. The cloth becomes negatively charged as it gains electrons.
4. True – Friction can cause the movement of electrons, so this is true.

→What is static electricity?


Static electricity is a type of electricity that is created by the buildup of electric charges on the surface of a
material. This buildup occurs when two materials are rubbed together, causing electrons to transfer from
one material to the other, creating an excess of electrons on one material and a shortage of electrons on
the other.
→How does static electricity work?
Static electricity works by the transfer of electrons from one material to another. When two materials are
rubbed together, electrons can transfer from one material to the other, creating an electric charge. The
resulting buildup of electrons on one material creates a negatively charged material, while the shortage of
electrons on the other material creates a positively charged material.
→What are some real-life applications of static electricity?
Static electricity has many real-life applications, including in electrostatic spray painting, electrostatic
precipitation, and electrostatic induction. It is also used in the production of computer chips, as well as in
electrostatic discharge (ESD) protection for sensitive electronic equipment.
→What are some dangers of static electricity?
While static electricity can be useful in some applications, it can also pose a danger in certain situations.
For example, static electricity can cause sparks that can ignite flammable materials or cause explosions. It
can also cause damage to sensitive electronic equipment, such as computers and other electronic devices.
→How can static electricity be prevented?
Static electricity can be prevented by using materials that are conductive or by using humidifiers to
increase the humidity in the air. Another method is to ground the materials that are producing the static
electricity, which allows the excess electrons to flow away from the material and prevent the buildup of
static charges.
→What are some interesting experiments that can be done to demonstrate static
electricity?
Some interesting experiments that can demonstrate static electricity include rubbing a balloon on your
hair and sticking it to a wall, using a Van de Graaff generator to generate high voltage static electricity,
and creating sparks by bringing two materials with different electric charges close to each other.

These questions and answers provide a comprehensive overview of static electricity, making it
easier for 15-16 year old students to understand the concept as they prepare for their GCSE
physics exams.

Charged Objects
Electrical Charges
Charge is a property of matter. Atoms make up all matter and consist of three sub-atomic
particles: protons, neutrons and electrons. Protons carry a positive charge and electrons carry a
negative charge.

Forces Exerted by Charged Objects


 Forces are exerted by charged objects. Two electrically charged objects can exert a force on each
other without directly touching. This force is an example of a non-contact force.
 Forces can attract and repel. Two objects with the same type of charge will repel, whilst two
oppositely charged objects attract. The force between unlike charges is called electrostatic attraction.
The force between like charges is called electrostatic repulsion.

 Electrostatic forces can cause movement. Similar to other forces, electrostatic forces can
cause movement. Electrostatic attraction causes objects to move closer together, whilst electrostatic
repulsion causes objects to move further away from each other. If we think back to our example with
the rod and the cloth, the positive rod will attract the negative cloth, so they will move closer together.

Conductors and Insulators


Conductors

Conductors easily allow the free movement of electrons inside of them. Metals are generally
good conductors of electricity as they have many free electrons inside them, which can travel
between atoms.

Insulators

Insulators do not contain free electrons, so are poor conductors of electricity.


Charged Objects

Dangers of Electricity
Dangers of Electricity
Dangers of the Live Wire
 The live wire is very dangerous. The live wire has an alternating pd of 230V from the mains supply,
making it very dangerous. The value of 230V is very far from the value of 0V, which is the normal
voltage of the human body.
 The live wire causes shocks. Since the live wire has a very large potential difference compared to the
human body, the live wire can cause shocks. When you touch a live wire, current flows through your
body and causes an electric shock.
 Open circuits are still dangerous. Even if the switch is open in a circuit (i.e. the circuit is not
complete), the circuit may not actually be safe. There might not be a current in the live wire, but there
will still be a potential difference in it. When touched, a current can flow from the live wire through
your body and into the earth.

Other Dangers of Electricity


 Water is dangerous around electricity. Water can conduct electricity, which means that it can provide
a path for electricity to flow to your body. This can lead to electric shocks, and therefore it is essential
to use dry hands when around electricity and when handling sockets Electricity should never be
handled in damp conditions!
 Fuses are important for safety. Sometimes electric circuits become overheated, and the amount of
electricity flowing through is very large. This can lead to the circuit becoming very hot, which can
potentially cause a fire. Fuses are devices which shut down the circuit if it the current becomes too
high. Similarly circuit breakers break a circuit if the current is too high.
 Sockets and wires have insulating coating. Sockets have a plastic insulating coating, which prevents
you from getting in direct contact with the mains supply. Wires also have this, but sometimes wires can
become cut and frayed, exposing the dangerous live wire. This can increase the risk of
being electrocuted.

More on Electrical Safety


Double Insulation

Appliances with double insulation do not have an earth wire. They only have a live wire and
neutral wire.

Dangers of Electricity

The whole appliance is insulated by plastic, meaning that no live conductor can touch the outer
casing. This can be seen in TVs and hairdryers.

Earthing

One safety measure is earthing.

As we mentioned, the live wire can cause electric shocks. To prevent this, the earth wire is
connected a metal casing. This means the current will pass through the earth wire instead of the
other components and prevent an electric shock from happening.

Dangers of Electricity
A very large current can flow through the metal case as it has a low resistance. The large current
would blow the fuse in the plug, disconnecting the appliance from the main power supply.

This can be seen in cookers and refrigerators.

→What is electricity and why is it dangerous?


Electricity is a flow of charged particles, usually electrons, that can power and run electrical devices.
However, it can also be dangerous if not handled properly. Electricity can cause shocks, fires, and even
death if it comes into contact with a person’s body or if it is used improperly.
→How does electricity cause shocks?
Electricity can cause shocks if it flows through the body, disrupting the normal electrical signals that the
body uses to communicate and control its functions. This can lead to muscle contractions, heart
arrhythmias, and in severe cases, death.
→Can electricity start fires?
Yes, electricity can start fires if it is not properly controlled or if there are defects in electrical wiring. An
electrical malfunction can cause sparks, heat, and flames, which can quickly spread and cause a fire.
→What should I do if I receive an electric shock?
If you receive an electric shock, you should immediately remove yourself from the source of the shock
and seek medical attention. If the person is unconscious, you should call 911 and perform CPR if you are
trained to do so.
→How can I prevent electric shocks?
To prevent electric shocks, it is important to follow safety guidelines and proper electrical safety
practices. This includes using electrical devices and appliances properly, avoiding water near electrical
outlets, and having regular electrical inspections to ensure that your home’s wiring is safe.
→What should I do if I see sparks or smoke coming from an electrical device?
If you see sparks or smoke coming from an electrical device, you should immediately unplug the device
and turn off the power source. Do not touch the device or try to repair it yourself. Call a licensed
electrician to inspect and repair the problem.
→What are some common dangers associated with electricity?
Common dangers associated with electricity include electric shocks, fires, and electrical burns. It is also
important to be aware of the dangers of electric arc flashes, which can cause serious burns and injuries.
→How can I protect myself from electrical hazards?
To protect yourself from electrical hazards, it is important to follow electrical safety guidelines, such as
avoiding water near electrical outlets, using electrical devices and appliances properly, and having regular
electrical inspections to ensure that your home’s wiring is safe. Additionally, it is important to be aware
of electrical hazards and to take steps to avoid them.

Power: Work Done


Work Done
Work is done when charge flows in a circuit.
As with any form of ‘work’, energy is transferred when electrical work is done. We will
calculate the energy transferred by electrical work in the next tutorial.

We can calculate energy transferred in one of two ways.

Using Power and Time

Power tells us the energy transferred per second. To find the total energy transferred over a
sustained period of time, we need to multiply power by the total number of seconds.

Where:

 energy transferred, E, in joules, J


 power, P, in watts, W
 time, t, in seconds, s

Question: An electric lamp transforms 400 J in 8 s. What is its power?

1. Write out the equation.

In this instance, we need to rearrange the equation to make power the subject.

E = Pt
P=E/t

2. Substitute in the numbers.

P = 400 / 8
P = 50 Watts

Question: A motor has a power of 12W. How much energy does the motor transfer in 1 minute?
E = P t = 12 x 60 = 720J

Question: Ankit is deciding between purchasing a lamp from Aryaland and a lamp from Makam
Bulbs. The lamp from Aryaland has a power of 6W, whilst the lamp from Makam Bulbs has a
power of 10W.

Assuming 100% efficiency in both lamps, what is the difference in the kilojoules (kJ) of energy
produced in 2 hours?

The difference between the powers: 10W – 6W = 4W

We need the answer in seconds, hence 2 hours = 120 minutes x 60


seconds = 7200 seconds

E = P x T = 4 x 7200 = 28800

To convert to kilojoules: 28800/1000 = 28.8

Using Charge and Voltage

We can also calculate energy transferring using charge and potential difference. Let us look at
the first equation, E = Pt.

We can substitute in P = VI into the equation E = Pt, to give E = VIt.

Current is just the rate of charge flow, I = Q/t. We can substitute in I = Q/t into E = VIt, to give E
= V x Q/t x t. This simplifies to E = VQ.
Where:

 energy transferred, E, in joules, J


 charge flow, Q, in coulombs, C
 potential difference, V, in volts, V

Question: A motor has a power of 12W. How much energy does the motor transfer in 1 minute?

1. Write out the equation.

E = QV

2. Convert into seconds.


In the question, we have been given a time in minutes, therefore we need to convert into seconds.

1 minute = 60 seconds

3. Substitute in the numbers.


We need to use the value for time in seconds (which we have just calculated).

E = 12 x 60
E = 720 J

Power Ratings and Energy


 The power rating is a maximum value. When a customer buys an electrical device, it comes with
a power rating. This is the maximum value at which the appliance can be used safely. We also know
that power is the rate of energy transfer. Therefore, the power rating also tells us the maximum rate at
which energy can be transferred when using the appliance.
 Power ratings can vary. Some power ratings are low, whilst others are high. We know that electrical
device can store energy. A high power device will be transfer more of this energy per second, therefore
doing more work. By doing so, a high power device will require more electricity.
Power: Current and Potential Difference
Power
The power of an appliance is the amount of energy (J) that it transfers per second.

In other words, power is the rate of energy transfer.

Power is measured in Watts (W).

1W means 1J of energy transferred in 1 second.

Calculating Power
Using Current and Potential Difference

We can calculate power in one of two ways. Firstly, we can use potential difference and current.

Remember, current tells us the rate of movement of charge. Whilst potential difference tells us
the energy transferred per unit charge. Therefore to find the total energy transferred per second,
power, you need to multiply current by voltage.

Where:

 power, P, in watts, W
 potential difference, V, in volts, V
 current, I, in amperes, A

Power is measured in Watts (W). 1W means 1J of energy transferred in 1 second.

Question: Billy is using a lamp. The current in the lamp is 0.4A, and the potential difference
across the lamp is 5V. Calculate the power supplied to the lamp.
1. Write out the appropriate equation.

P = VI

2. Substitute in the numbers.

P = 5 x 0.4
P=2W

Question: There is just one bulb and one cell in a series circuit. The cell provides a voltage of
6V. A current of 3A flows in the circuit. What is the power of the bulb?

The bulb is the only component, so gets the maximum voltage of 6V.

Power = Current x Voltage = 3 x 6 = 18W

Question: An electric fire has a power of 1.25kW and a potential difference of 210V. Fuses are
available in the standard ratings of 2A, 5A and 10A. Which would be the best fuse to use?

1. Write out and rearrange the appropriate equation.

P = VI
I=P/V

2. Calculate the current.

I=P/V
I = 1250 / 210 = 5.9523
I = 5.95 A

3. Determine which fuse to use.

Now we have calculated the current, we can see that the best
fuse to use would be the 10A fuse as the other fuses would blow
as their values are too low.

→What is Power?
Power is the rate at which energy is transferred. It is a measure of how much work is being done in a
certain amount of time. Power is often expressed in units of watts (W).
→What is the formula for Power?
The formula for power is P = IV, where P is power, I is current, and V is voltage or potential difference.
→What is Current?
Current is the flow of electric charge. It is measured in units of amperes (A). The flow of current is like
the flow of water in a river. Just as a river can have a strong or weak flow, an electrical current can be
strong or weak.
→What is Potential Difference?
Potential difference, also known as voltage, is the force that drives the flow of electric charge. It is
measured in units of volts (V). The potential difference between two points is like the height difference
between two points in a river. The greater the height difference, the greater the flow of water. Similarly,
the greater the potential difference, the greater the flow of electric charge.
→What is the relationship between Current, Potential Difference and Power?
Current and potential difference are related to power in that power is equal to the product of current and
potential difference. The more current flowing through a circuit, and the greater the potential difference,
the more power is transferred.
→How does Power affect energy transfer?
The amount of energy transferred depends on both the power and the amount of time for which the
energy is transferred. The higher the power, the more energy is transferred in a given amount of time.
→Can you give an example of Power in everyday life?
An example of power in everyday life is a light bulb. The higher the power rating of a light bulb, the
brighter it will be. The amount of energy used by a light bulb depends on both the power rating and the
amount of time for which it is used.
→Why is it important to understand Power, Current and Potential Difference?
Understanding power, current, and potential difference is important because they are fundamental
concepts in electricity and electronics. These concepts are used in many everyday appliances and in many
industries, including construction, manufacturing, and telecommunications. By understanding these
concepts, you can better understand how electrical devices and systems work.

Series and Parallel Circuits


What is the Difference Between Series and Parallel Circuits?
There are two main types of circuits:

 Series circuits – in a series circuit, all components are connected in line with each other.
 Parallel circuits – in parallel circuits, the components are connected in separate loops.

Some circuits can have both series and parallel parts.


Series and Parallel Circuits

Series and Parallel


Circuits

Series Circuits

There are a few rules and facts about series circuits that we need to remember.

 Current is the same in all parts of the circuit. Current is not ‘used up’ along the circuit, so
remains constant. You can calculate the current by dividing the total voltage of the battery or cell by
the total resistance in the circuit.
 Total voltage is shared amongst components. The total voltage from the battery / cell
is shared between the components – if the battery or cell provides 5V of energy, and there are two
lamps (with equal resistance), each lamp will get 2.5V.
 Total resistance is the sum of the resistances of all components. Current needs to be the same
everywhere, so the higher the resistance of an individual component, the greater its share of
the voltage (so that current, which is voltage / resistance, stays equal). Therefore if you add an extra
lamp to a series circuit, the total resistance increases and hence the current decreases. All lamps become
dimmer. We can use the following equation to work out the total resistance in a series circuit:
Where:

 Resistance, R, in ohms, Ω

 Cell voltages sum together. If there are two cells or batteries, then the voltage from both add
up together to give the total circuit voltage.
 A break in one component ruins the whole circuit. There is only one route for charge, so if one
lamp in a series circuit is broken, all lamps will stop working. Christmas lights are often sometimes in
series circuits, because each bulb only needs a small voltage, so it is better to share the voltage in series.
However, this means that if one light breaks then they all break.
Series and Parallel
Circuits

Parallel Circuits

The voltage is the same for all components. In series circuits the voltage was shared between
the components (in proportion of their resistance). In parallel circuits, every single component
gets the full, maximum voltage. Therefore bulbs in parallel will have the same brightness
(assuming equal resistance). This means more components may be added in parallel without
needing extra voltage.

Series and Parallel Circuits


 The current is split between the branches of the circuit. The sum of the currents of all branches is
equal to the total current that flows from the cell / battery. In other words, the total current through the
whole circuit is the sum of the currents through the separate components.

Series and Parallel Circuits

 A break in one component does not ruin the whole circuit. There is more than one
route for charge, so if one lamp in a parallel circuit is broken, only the lamps in the broken branch are
affected. Lighting in houses is often connected in parallel because it means that we can control different
branches of the circuit (representing different rooms) using different switches.

Series and Parallel Circuits

 Total resistance is worked out using the reciprocal of the resistances of each branch. If there are 3
branches to the circuit (R1 – R3), then 1 / Total Resistance = 1 / R1 + 1 / R2 + 1 / R3. This means that
the total resistance of two resistors is less than the resistance of the smallest individual resistor.
Where:

 Resistance, R, in ohms, Ω

Resistors
 Resistors in series increase resistance. In a series circuit, adding resistors will increase resistance. The
current is the same in every single component in the circuit, so the more resistors we add, the harder it
is for current to flow. This means that the overall resistance has increased in a series circuit.
o The more resistors added, the harder it is for current to flow.
 Resistors in parallel decrease resistance. In a parallel circuit, adding resistors will decrease the
resistance. The potential difference is the same in every single component in the circuit. The more
resistors we add in parallel, the more ‘pathways’ the current has to go through, so it is easier for current
to flow through the circuit. This means that the overall resistance has decreased in the circuit. The more
resistors added, the easier it is for current to flow through many pathways.
o The more resistors added, the easier it is for current to flow through many pathways.

Calculating Resistance in Series

For exams, we need to be able to calculate the combined resistance, also called the equivalent
resistance, of resistors.

Question: In the diagram below, there are two resistors in series. R1 has a value of 20 ohms,
whilst R2 has a resistance of 10 ohms. Find the equivalent resistance of the two resistors.

We need to remember that the equivalent resistance simply means the total of the resistors in the
circuit.

R total = R1 + R2
R total = 20 + 10
R total = 30 Ohms
Summary Comparison: Series vs. Parallel Circuits

Series and Parallel


Circuits

Investigating Resistance in Experiments


Experiments with Series Circuits

For exams, we need to design and use dc series circuits. We can use these circuits to investigate
resistors.
Method:

1. Set up circuit. We have to set up a circuit with a cell (battery), an ammeter and a resistor in series.
2. Check the circuit. Make sure that the circuit is complete and working.
3. Measure PD. Record the potential difference of the cell, in volts.
4. Measure current. Measure the current in the circuit using an ammeter, recording the value in amps.
5. Calculate resistance. Calculate the resistance in the circuit, by rearranging V = IR. Record the value
for resistance in ohms.
6. Change quantity of resistors. Add another resistor to the circuit and then repeat steps 2-5.
7. Fill in a table. The results table should look something like this:

8. Plot a graph. Using the results, we can make a graph of ‘quantity of resistors’ (x axis) against ‘total
resistance’ (y axis).

Experiments with Parallel Circuits

You can also investigate resistors in parallel. The experimental process is largely the same.

In exams, this is an example of application questions. You should have learnt the experimental
set up for series, so they want to see you apply your same knowledge to a less familiar scenario
(in parallel)

Short Circuits
Series and Parallel Circuits

Current likes to take the easiest path with the least resistance. In this diagram, if switches P and
Q are closed in this circuit, then neither lamps will be lit. The current prefers to pass through
plain wire than the lamps, because plain wire has a lower resistance. Therefore a short circuit is
formed.

When the switches are open, the current is forced to go through the bulb, and so the bulb lights
up.

For this kind of circuit, there is no parallel split in current. If the plain wire path is available, then
all of the current will go down that pathway.

→What are series circuits?


A series circuit is a type of electrical circuit in which the components are connected in a line, one after the
other, so that the same current flows through all of them.
→What are parallel circuits?
A parallel circuit is a type of electrical circuit in which the components are connected so that each
component has its own separate branch and the same voltage is applied to each component.
→How do series circuits differ from parallel circuits?
In a series circuit, the components are connected in a line and the same current flows through all of them.
In a parallel circuit, the components are connected so that each component has its own separate branch
and the same voltage is applied to each component.
→What happens to the total resistance in a series circuit?
In a series circuit, the total resistance is equal to the sum of the resistance values of the individual
components.
→What happens to the total resistance in a parallel circuit?
In a parallel circuit, the total resistance is lower than the resistance of any of the individual components.
→What happens to the total current in a series circuit?
In a series circuit, the total current is the same as the current in any of the individual components.
→What happens to the total current in a parallel circuit?
In a parallel circuit, the total current is equal to the sum of the current in each of the individual branches.
→Why are parallel circuits used more often than series circuits in everyday life?
Parallel circuits are used more often than series circuits because they allow multiple components to be
powered by the same voltage source while still maintaining separate branches for each component. This
allows for more flexibility in the design and usage of electrical circuits.
Non-renewable Energy Sources
Non-renewable Energy Resources
Non-renewable energy resources include fossil fuels (coal, oil and natural gas) and nuclear fuel
(uranium and plutonium).

Characteristics of Non-Renewable Energy Resources


 Non-renewable resources may run out. Fossil fuels are natural resources formed from the living
remains of organisms millions of years ago. However, fossil fuels are in limited supply so they will
eventually run out.
 Non-renewable resources are reliable. They are readily available and easy to obtain unlike renewable
resources such as solar energy, which is dependent on the weather.
 Non-renewable resources are relatively cheap. Fossil fuel power plants are quite cheap to build and
manage. The cost to extract fossil fuels is also relatively low.
Environmental Impact
There are several issues caused by non-renewable resources:

 Greenhouse gases – as previously discussed, burning fossil fuels cause to the environment through the
production of greenhouse gases. When burnt, non renewable resources release carbon dioxide into the
atmosphere. This CO2 contributes to global warming.
 Acid rain – also through the burning of fossil fuels, we can produce sulphur dioxide, which causes
acid rain. The SO2 gas is released into the atmosphere, mixes with rain and produces acid rain. This
acid rain
can damage buildings, kill fish in lakes and also be harmful to trees.

 Radioactive waste – when we use nuclear fuels, we create extremely harmful radioactive waste. This
waste is very difficult to get rid of and causes damage to the environment and humans.
 Oil spillages – accidentally, tankers, barges and pipelines can leak into the ocean, causing oil spillages.
These spillages of oil will pollute the oceans and harm marine life.

Although we know about these environmental effects of using certain energy resources, we
cannot always combat these issues. This is because of the following considerations:
 Political – sometimes governments try to introduce charges and taxes to discourage people from doing
certain things. For example there is a Congestion Charge in London, where drivers have to pay a fee to
be able to drive in a particular area of the capital at certain times. By doing this, the government hoped
to decrease the pollution in the capital from fossil fuels.
 Social – although solar panels and wind turbines are good renewable energy resources, some people
don’t want panels on their roof, or a turbine plant in their back garden. These are forms of
‘visual pollution’, which spoil an otherwise aesthetically pleasing view.
 Ethical – when we use nuclear fuels, we produce lots of radioactive waste. This waste is extremely
harmful and stays dangerous for hundreds of years. It can be argued that it is unethical to create this
type of waste, since it has a tremendous impact on future generations.
 Economical – many renewable energy sources involve a large amount of money to set up. For
example, building solar panels, wind turbines and cars that run on biofuels are all
incredibly expensive to make. We
have to consider whether all these start up costs will be beneficial in the long run.

Energy Resources: Electricity Generation


Generating Electricity
So far we’ve looked at how energy resources can be used in heating and transport. Energy
resources can also be used to generate electricity.

Electricity can be generated using renewable or non-renewable energy resources.

Non-Renewable Energy Resources

Here are some examples of non-renewable energy resources used to generate electricity:

 Thermal power stations. Heat energy is used to turn water into steam. The steam drives a turbine,
which then drives a generator to produce electrical energy. Non-renewable energy sources may be
used as the energy source to heat the water, such as coal which is a fossil fuel. Another example is the
use of nuclear fuels, for example uranium or plutonium.
Energy Resources – Electricity
Generation Pages

 Gas-fired power stations. Similar to thermal power stations, but natural gas is burnt in a gas turbine
which is directly linked to the electricity generator.

In thermal power stations, cooling towers are used to condense the steam from the turbine, back
into water. Therefore heat energy is lost from the cooling towers, which is a waste of energy.
This means that thermal power stations have an overall efficiency of around only 30%. Gas-fired
power stations are more efficient than thermal power stations. The hot gases released by the
turbine are used to produce more steam. The steam is then used to generate more electricity
using a steam turbine and a separate generator. This means that less energy is lost overall.

Renewable Energy Resources

As we explained in a previous tutorial, renewable energy resources such as hydroelectricity,


wind, wave, tidal and geothermal resources are used to generate electricity.

Here are a few examples of renewable energy resources used to generate electricity in a bit more
detail:

 Solar Power. Solar energy from the Sun can be used to generate electricity. Solar panels convert light
into heat energy. A solar furnace can produce heat energy, which can then be used to convert water
into to steam. The steam turns a turbine to produce electricity using a generator.
 Wind Turbines. These work similarly to solar power. Wind causes the blades to rotate on a wind
turbine. The rotating blades will then turn a generator which will generate electricity. Wind turbines
supply a huge amount of electricity, so are very cost effective.
Energy Resources –
Electricity Generation Pages

Some of the causes of energy loss seen with non-renewable energy resources, such as water
cooling towers, are not used with renewable energy resources. This means that generating
electricity using renewable energy resources is usually more efficient and cost effective.

Renewable Energy Sources


Energy Resources
The following table shows the main energy resources available for us to use on Earth.

The percentages show how much each resource contributes to our total energy usage. The
resources without a percentage make up a very small amount of our overall energy usage. The
types of energy resources can be split into non-renewable and renewable energy resources.
Renewable Energy Sources

Renewable Energy Sources

Renewable Energy Resources


 Renewable energy can be replenished. Renewable energy resources are replenished as they are used
up (or they will be replenished later). One advantage of renewable energy is that we will never run out
of these resources.
 Renewable energy doesn’t damage the environment. Another advantage of renewable energy
resources is that they don’t cause much damage to the environment. This is much better than
nonrenewable resources, which produce greenhouse gases that are quite harmful to the environment.
 Renewable energy isn’t very reliable. However, renewable energy resources are not always the most
reliable forms of energy. As you can see from the table, a lot of renewable energy resources depend on
the weather, such as the Sun, wind and waves. These are quite unpredictable, meaning that we might
not always get a steady supply of energy when we need
Efficiency
Efficiency
We previously mentioned that not all the energy from a transfer is converted into useful energy.
Instead, some of the energy will be converted into wasted energy, which is of no use to us.

To make energy transfers more efficient, we have to maximise the useful energy and minimise
the wasted energy from a transfer.

Calculating Efficiency
We can calculate efficiency in one of two ways. In this tutorial we will focus on efficiency in
terms of energy. In both cases, efficiency is measured as a ratio, so the units will cancel out.

Efficiency in Terms of Energy

Efficiency

Where:

 Energy is measured in joules, J


 Efficiency has no units

Question: Megan is using her hair dryer. It wastes 376J of every 1500J of electrical energy it
uses up. How efficient is Megan’s hairdryer, to 2dp?

1. Write out the equation.


In this case, we are dealing with energy so the appropriate equation is:Efficiency = useful output energy
transfer / total input energy transfer
2. Work out the useful output energy.
In the question, we have only been told how much energy the hairdryer wastes, not how much is
converted into useful energy.
Efficiency

3. Substitute in the numbers.


Now that we know the numbers, we can simply put them into the equation.

Efficiency

Percentage of Efficiency

The percentage of efficiency can be calculated by multiplying the efficiency by 100.

Efficiency

Question: A LED bulb is supplied a total of 135J. Out of the total energy supplied, 102J is
usefully transferred. How efficient is the bulb, expressed as a percentage to 2 dp?

1. Write out the equation.


Again, we are dealing with energy so the appropriate equation is:Efficiency = useful output energy
transfer / total input energy transfer
2. Substitute in the numbers.
We know the numbers from the question, so we can simply put them into the equation.

Efficiency

3. Express as a percentage.
Simply multiply by 100 to work out the percentage of efficiency.
Efficiency

Drawing Diagrams to Show Efficiency


Sankey Diagrams

Seen we know that most devices are not 100% efficient, we can draw diagrams to visualise how
much energy is actually useful.

This is good for comparing efficiency, for example in lightbulbs. We most commonly
use Sankey diagrams, where the thickness of the arrows show how much energy is being
transferred.

Efficiency

The diagrams comparing a filament light bulb and an energy saving light bulb, show us that
more energy is wasted as heat in the filament light bulb.
Efficiency

Reducing Energy Waste


Increasing Efficiency
Efficiency

 We can increase efficiency in several ways. We can increase the efficiency of


energy transfers in a few different ways:
thermal insulation, lubrication and streamlining.
 100% efficiency is usually impossible. It is normally impossible for an energy
transfer to be 100% efficient. There is usually always some wasted heat energy in
the form of heat / thermal energy. Sometimes the definition of ‘useful’ energy can
change – e.g. some lights are designed to provide heat and light, so the heat
energy is also useful!

Reducing Unwanted Energy Transfers

There are several ways in which we can reduce the amount of unwanted energy transfers:

1. Thermal insulation. A common way in which energy is wasted is


through thermal energy. For example, when a house is heated, energy can be
transferred through the windows, doors and walls. In order to combat this, houses
are often insulated and windows are double glazed.

Reducing Energy Waste

2. Lubrication. Energy can be lost due to friction. When you ride a bike, the chain
will experience a lot of friction. In order to combat this, you can put oil on the
chain. This will stop the chain experiencing so much friction and will maintain the

bike in the long run. Reducing


Energy Waste

3. Streamlining. When objects travel through the air, they experience air
resistance. In order to combat this, we streamline objects. Streamlining is when
we change the shape of an object so that it doesn’t feel the effects of air resistance
as much. For example, airplanes have ‘pointy’ noses at the front so that they can
travel through the air at high speeds without experiencing too much resistance.

Reducing Energy Waste

Cooling a Building

Our homes are great examples are where we use thermal insulators.

When we heat up our homes, we want them to stay warm for as long as possible. However, the
thermal conductivity of the doors, windows and walls can result in energy losses. In order to
lessen the amount of thermal energy that escapes from our homes, we need to reduce thermal
conductivity. This can be achieved in the following ways:

1. Double glazed windows – by using double glazing, we are thickening the


windows and therefore reducing their thermal conductivity. As the thermal
conductivity decreases, the rate of thermal energy transfer will also decrease.

Reducing Energy Waste


2. Insulated walls – the walls of a house can have a very high thermal conductivity,
meaning lots of thermal energy transfer to the surroundings. To reduce the
amount of thermal energy transfer, we can put an air gap in between the layers.
To reduce the transfer further, we can fill the gap with foam. Both of these actions
will increase the thickness of the walls, therefore decreasing the thermal

conductivity. Reducing Energy Waste

3. Loft insulation – similar to the windows, lofts can transfer a lot of thermal
energy via thermal conductivity. To prevent this from happening, lofts are often
lined with fibreglass wool. This will insulate the loft and prevent energy losses.

Red
ucing Energy Waste

Investigating Thermal Insulators


Method

1. Gather the equipment. For this experiment, you will need a beaker (with a lid)
filled with hot water, a thermometer, a top pan balance, a stopwatch and different
insulating materials.
2. Measure the filled beaker’s mass. Place the beaker full of hot water onto a top
pan balance. Make sure that the lid is on top of the beaker too. Record the mass in
kilograms.
3. Measure the water temperature. Use the thermometer to measure the
temperature of the water in the beaker. Record this as the ‘initial temperature’,
with the units in °C.
4. Time for 5 minutes. Make sure that you have placed the lid on the beaker. Start
the stopwatch and time for five minutes.
5. Measure the water temperature. Once the five minutes are up, you can measure
the temperature of the water again. Record this as the ‘final temperature’, with the
units in °C.
6. Calculate the temperature change. To find the change in temperature, do the
calculation ‘initial temperature – final temperature’. Record this as the
‘temperature change’, with the units in °C.
7. Repeat steps 2-6 with different insulators. For each experiment, make sure that
you have the same mass of water and the same initial temperature. The only thing
that should change is the material that you wrap around the beaker.
8. Record your results in a table. The results table should look like this:

Reducing Energy
Waste

9. Interpret the results. From the table, we should be able to identify the
temperature change with each insulator. The bigger the change, the more thermal
energy transfer has occurred from the water to the surroundings. Therefore, the
best insulator will be the one with the least change in temperature.

→What is energy waste?


Energy waste refers to the loss of energy that occurs during the production, distribution, and use of
electricity, gas, and other forms of energy. This energy is often lost as heat, light, or sound, which can
contribute to environmental problems and increase energy costs.
→Why is reducing energy waste important?
Reducing energy waste is important for several reasons. Firstly, it can help to save money by reducing
energy bills. Secondly, it can help to reduce carbon emissions, which can contribute to climate change.
Finally, it can help to conserve natural resources, such as coal, gas, and oil, which are finite and will
eventually run out.
→What are some simple ways to reduce energy waste?
There are many simple ways to reduce energy waste, including:

Turning off lights and appliances when they are not in use
Using energy-efficient light bulbs Insulating your home to prevent heat loss
Using a smart thermostat to control your heating and cooling
Setting your computer to “sleep” mode when you are not using it
Unplugging chargers and other electronics when they are not in use
→How can I reduce energy waste in my home?
To reduce energy waste in your home, you can take several steps, including:

Installing low-flow showerheads and toilets to reduce water waste


Replacing old appliances with energy-efficient models
Sealing air leaks around windows and doors to prevent heat loss
Planting trees or installing shading devices to reduce solar heat gain in the summer
Insulating your attic, walls, and floor to keep your home warm in the winter and cool in the summer
→What is the impact of energy waste on the environment?
Energy waste can have a significant impact on the environment, as the energy that is lost often contributes
to greenhouse gas emissions, which can contribute to climate change. Additionally, the production and
transportation of energy sources, such as coal, gas, and oil, can lead to air and water pollution, habitat
destruction, and other environmental problems.
→How can schools and communities reduce energy waste?
Schools and communities can reduce energy waste by implementing energy-saving initiatives, such as:

Installing energy-efficient lighting and appliances


Encouraging the use of public transportation, biking, or walking instead of driving
Recycling and composting to reduce waste
Implementing an energy management program to monitor and reduce energy use
Hosting educational programs and events to raise awareness about the importance of reducing energy
waste

Energy Resources: Heating


Energy Resources
Energy resources can be used in heating and transport. They are necessary to create thermal
energy to heat up buildings or water.

Heating
 Natural Gas. Most often we use natural gas to heat up water which then runs through radiators in a
house. The gas is burned to heat up water in a water tank via conduction. The hot, less dense water
then rises via convection and travels to the radiator. The radiators then heat up the house via radiation.
As the water in the radiator loses heat to its surroundings, it becomes cooler and more dense, so it
travels back down to the water tank.

Energ
y Resources: Heating

 Renewable energy resources. If we don’t use natural gas, we can use renewable energy resources, such
as biofuels or solar panels (we aren’t making electricity, we will be using the energy to heat up
buildings or water). Infrared radiation from the sun is collected to heat up cold water. The hot water
can then be stored in a hot water storage tank for when we want to use hot water in our taps.
A boiler may be used to increase the temperature of the water.
Energy
Resources: Heating

Thermal Radiation
Radiation
Radiation involves transfer of heat via infra red waves, unlike thermal conduction and
convection which need the vibration or movement of particles.

This means radiation can occur in a vacuum, where there are no particles of matter. The sun
provides heat to the earth due to radiation.

An object can absorb radiation, causing the temperature of the object to rise. However it can also
partly reflect or transmit radiation.

Some surfaces are better at absorbing and emitting heat radiation than others. This can be
investigated using boiling tubes covered in different surface materials.

Investigating Thermal Radiation


Method
1. Gather the equipment. For this experiment, you will need boiling tubes, a kettle, test tube rack,
various paper of different materials, rubber bungs, a timer and a thermometer.
2. Assemble the equipment. Place the test tubes in a test tube rack. Cover each tube in the same area of
paper but of the following different surfaces.
o Black matt paper
o Black paper with a glossy surface
o White matt paper
o White paper with a glossy surface
3. Fill the tubes. Fill each tube with hot boiling water.
4. Check the temperature. Using the thermometer, check the temperature of the water in each tube and
record your results.
5. Start the timer. Lightly seal each tube with a rubber bung and start the timer.
6. Record the temperature as it decreases. Every two minutes remove the rubber bung and record the
temperature of the water.
7. Repeat. Repeat this process until ten minutes have passed and record your final temperature.
8. Record your results in a table. The results table should look like this:

Therm
al Radiation

9. Plot a graph. Draw a graph of temperature vs time.


10. Interpret the results. The experiment shows us that darker, matt surfaces are better absorbers and
emitters of infrared radiation. Lighter, shiny surfaces are poorer absorbers and emitters of infrared
radiation. This means they are good reflectors of infrared heat.

Examples of Thermal Radiation


 Clothing in hot weather. In countries with a warm climate, people are recommended to
wear white clothing instead of black. This means you’ll absorb less heat and stay cooler for longer.
 Foxes. Foxes with white fur are better adapted to survive in cold conditions compared to foxes with
brown fur. Their white fur reduces the amount of heat radiated, so they can retain more heat to survive
the cold weather.
Heat Transfer
Forms of Heat Transfer

We have now covered the main forms of heat transfer (conduction, convection and radiation).

Thermal Radiation

However, sometimes we want to reduce the amount of heat transfer. A great example to illustrate
this is a vacuum flask. A vacuum flask has many adaptions to reduce energy transfer via
conduction, convection and radiation.

Vacuum Flask
 Silvered surfaces. These surfaces reflect any infrared radiation back towards the hot liquid. The
silverly surface is also a poor emitter of radiation, so less heat is lost to the surroundings.
 Plastic stopper. Plastic is a poor conductor and insulator of heat, reducing thermal conduction. This
also stops convection currents from forming at the top of the flask, reducing energy transfer by
convection.
 Vacuum. The vacuum stops energy transfer by conduction and convection, as these processes require
the movement of particles to occur.
 Glass walls. Glass is also a poor conductor of heat, reducing energy transfer by conduction.

Thermal Radiation

→What is thermal radiation?


Thermal radiation is the transfer of heat energy through electromagnetic waves, which can travel through
a vacuum and do not require a material medium to transfer the energy.
→How does thermal radiation work?
Thermal radiation occurs due to the emission of electromagnetic waves from an object, which are caused
by the movement of charged particles within the object. These waves can be absorbed or reflected by
other objects, which can result in a transfer of heat energy.
→What are some examples of thermal radiation in everyday life?
Examples of thermal radiation in everyday life include: feeling the warmth of the sun on your skin, using
an infrared thermometer to measure the temperature of an object, and feeling the heat from a fire.
→What is the relationship between the temperature of an object and the amount of
thermal radiation it emits?
The amount of thermal radiation emitted by an object is directly proportional to the fourth power of its
temperature. This relationship is described by the Stefan-Boltzmann law.
→What is the relationship between the temperature of an object and the wavelength of the
thermal radiation it emits?
The wavelength of the thermal radiation emitted by an object is inversely proportional to its temperature.
This relationship is described by Wien’s law.
→What is an example of an object that emits thermal radiation?
All objects above absolute zero temperature emit thermal radiation, including the human body, a light
bulb, and a star.
→What is a black body?
A black body is an idealized object that absorbs all of the thermal radiation that falls on it, and emits
thermal radiation with a spectrum that depends only on its temperature. It is often used as a theoretical
model to describe the behavior of thermal radiation.
→What is an example of a material that is a good emitter of thermal radiation?
Materials that are good emitters of thermal radiation include black bodies, metals, and some non-metals
such as carbon.
→What is an example of a material that is a poor emitter of thermal radiation?
Materials that are poor emitters of thermal radiation include polished metals and some non-metals such as
glass.
→What is an example of a material that is a good absorber of thermal radiation?
Materials that are good absorbers of thermal radiation include black bodies and materials with a rough or
irregular surface.
→What is an example of a material that is a poor absorber of thermal radiation?
Materials that are poor absorbers of thermal radiation include polished metals and materials with a
smooth or shiny surface.

Thermal Convection
Convection
Heat is usually transferred by conduction in solids. In liquids and gasses, convection and
radiation are more important.

Convection is the flow of heat through a fluid from areas of a higher temperature to areas of a
lower temperature by movement of the fluid itself.

A convection current is a stream of warm moving fluid.

Convection in liquids

Convection of a liquid can be seen in an experiment using potassium permanganate.

A crystal of potassium permanganate is placed in a beaker of water and gently heated with a
Bunsen flame. Heat will be transferred through the glass wall of the beaker by conduction. This
heats the water near the Bunsen flame, so it becomes less dense and rises. The cold, denser air
above it sinks down and is in turn heated by the Bunsen flame. The pink colour due to the
potassium permanganate shows the convection current.
Thermal Convection

Convection in air

Radiators transfer heat via convection. Air near the radiator is heated, which causes it to become
less dense. Therefore this warm, less dense air rises up the room, and the cold, denser air above
it sinks down. The cold air can now be heated by the radiator. This cycle goes on and on, and
a convection current is formed. The heat is spreading through the room via this current.

Thermal Convection

Natural Convection Currents


 Coastal breezes – during the day, the temperature of land increases at a faster rate than the sea. The hot
air above the land rises and is replaced by colder air from the sea, resulting in a breeze from the sea.
 Gliding – gliders use hot air currents, known as thermals, to fly for several hours.

Thermal Conduction
Thermal Conductivity
Conduction

Conduction is the flow of thermal energy through matters of higher temperature to matters of
lower temperature.

The Transfer of Thermal Energy


 Thermal energy is transferable. Thermal energy can be transferred between objects, or from an
object to its surroundings. For example, when we heat up soup in a saucepan, thermal energy will be
transferred from the pan to the soup, then from the soup to the surroundings.

Thermal Conduction

 Conduction is primarily in solids. Conduction is the main form of heat transfer in solids. This is
because the particles are close by and can pass on vibrations easily, which is harder in a gas. Heat in
liquids and gases is normally transferred by convection and radiation, which we cover in later
tutorials.
 The rate of transfer depends on thermal conductivity. Thermal conductivity will determine
how fast the thermal energy is transferred. During conduction, vibrating particles transfer heat energy
to adjacent particles, and the vibration passes along, leading to the spread of heat. The higher the
thermal conductivity of an object, the higher the rate of thermal energy transfer.
Conductors and Insulators of Heat
There are good conductors and bad conductors (insulators) of heat. We will look at some
examples of both.

Good Conductors of Heat

These are helpful when heat needs to be quickly transferred. Most metals are
good conductors of heat. Here are some examples:

 Silver – you can find spoons made of silver. If use a spoon to eat some hot soup, heat will be
transferred to the spoon from the soup and it will become very hot.
 Aluminium – some saucepans are made of aluminium, allowing the food in the saucepan to get heated
quickly.
 Iron – when we iron a shirt on an ironing board, heat from the shirt is conducted to the shirt to remove
any creases.

Insulators of Heat

Insulators are often used when we want to reduce unwanted heat loss. Here are some examples
of insulators:

 Air – air is a very bad conductor of heat. This explains why we find it in between the two panes of
glass in double-glazed windows, as less heat will be lost.
 Wood – we often see the handles of saucepans made of wood. This means less heat will be transferred
from the metal pan to the handle, so we don’t burn our hands when we hold the handle.
 Plastic – found on the handle of an electric kettle. So when you boil water to make a cup of tea, the
plastic acts as an insulator so you won’t burn yourself when pouring the water into your mug.
 Wool – insulators like wool can trap air to reduce heat loss, for example in fleece winter jackets.

Investigating Thermal Conductivity


Thermal Conduction

Method
1. Gather the equipment. For this experiment, you will need a bunsen burner, a tripod, four rods made of
different types of metal, a bench mat, a stopwatch, four matches and some paraffin wax.
2. Assemble the equipment. Attach each match to a metal rod using an equal amount of paraffin wax.
Place the rods over the tripod at an equal distant from each other.
3. Turn on the bunsen burner. Set the bunsen burner to a low flame.
4. Start the timer and heat. Bring the bunsen burner to the tip of the rods and start the timer.
5. Record the time as each match drops off the rod. As the temperature of the far end reaches the
melting point of the wax, the matches should drop off.
6. Repeat. Repeat steps 2-5 twice more and calculate an average time for each type of metal rod. Ensure
the length of the metal rods is the same.
7. Record your results in a table. The results table should look like this:

Therm
al Conduction

8. Interpret the results. The rod which has the match that falls off first,
shows that thermal energy was transferred through the rod the fastest. Therefore, it is the best conductor
and worst insulator.
FAQs
→What is thermal conduction?
Thermal conduction is the transfer of heat energy through a material without any flow of the material
itself.
→How does thermal conduction work?
Thermal conduction occurs due to the movement of free electrons and vibrating atoms within a material.
Heat energy is transferred from the hotter end to the cooler end of the material as a result of this
movement.
→What are some examples of thermal conduction in everyday life?
Examples of thermal conduction in everyday life include: the handle of a hot pan heating up, a metal
spoon left in a cup of hot tea becoming hot to the touch, and a metal roof becoming hot on a sunny day.
→How does the thickness of a material affect thermal conduction?
Thicker materials generally have a lower thermal conductivity, which means they are less efficient at
transferring heat energy.
→What is thermal conductivity?
Thermal conductivity is the measure of a material’s ability to transfer heat energy. It is measured in watts
per metre per Kelvin (W/mK).
→What factors affect thermal conductivity?
The factors that affect thermal conductivity include: the type of material, the temperature of the material,
the density of the material, and the presence of impurities or defects in the material.
→What is a thermal insulator?
A thermal insulator is a material that has a low thermal conductivity, which means it is efficient at
preventing heat energy from being transferred through it.
→What are some examples of thermal insulators?
Examples of thermal insulators include: fiberglass, wool, Styrofoam, and air.
→What is a thermal conductor?
A thermal conductor is a material that has a high thermal conductivity, which means it is efficient at
transferring heat energy through it.
→What are some examples of thermal conductors?
Examples of thermal conductors include: metals such as copper and aluminum, and some non-metals such
as graphite.
→How does the surface area of a material affect thermal conduction?
The larger the surface area of a material, the more heat energy it can transfer. This is because a larger
surface area allows for more free electrons and vibrating atoms to be in contact with each other, resulting
in a higher rate of heat transfer.

The Law of Conservation of Energy


Conservation of Energy
The conservation of energy is an important principle in Physics. According to this principle, we
can’t ‘lose’ or ‘gain’ energy:
Energy can be transferred usefully, stored or dissipated, but cannot be created or destroyed.

The Law of Conservation of Energy

Since we know that energy cannot be created or destroyed, all the energy from a system must
be dissipated somehow. The dissipated energy can be useful, or not useful (e.g. wasted energy).

In many energy transfers, thermal energy is a waste product. For example, when we switch on a
light bulb, chemical energy is being transferred to light energy, but some of the energy will be
‘wasted’ as thermal energy.

Energy Transfers in a Closed System


 Closed systems don’t exchange with their surroundings. Previously, we mentioned that closed
systems are unable to exchange energy to matter with their surroundings. For example, a thermos flask
is a closed system as heat cannot escape (ignoring negligible amounts of heat loss).
 Energy transfers can occur in closed systems. Like any other system, energy can be transferred in a
close system. However, since energy cannot exchange with the surroundings, there will be no net
change to the total energy in a closed system.
 Adding ice cubes to a water bottle is an energy transfer. If you put ice cubes into a full water bottle
and close the lid, you are transferring energy. We are assuming that the water bottle doesn’t allow any
energy exchange with the surroundings, creating a closed system. The water will exchange thermal
energy with the ice cubes, so the water will cool down.

FAQs
→What is the law of conservation of energy in physics?
The law of conservation of energy states that energy cannot be created or destroyed, but it can only
change form. This means that the total amount of energy in a system remains constant, even as it is
transferred and transformed from one form to another.
→How does the law of conservation of energy apply to real-world situations?
The law of conservation of energy has a wide range of applications in the real world, including:

In mechanics, it is used to calculate the potential and kinetic energy of objects in motion.
In electrical circuits, it is used to calculate the energy transfer between components.
In thermodynamics, it is used to understand the relationships between temperature, heat, and work.
→What is the difference between potential energy and kinetic energy?
Potential energy is energy that is stored in an object due to its position or state. For example, a book on a
shelf has potential energy due to its height above the ground. Kinetic energy is energy that is in motion,
such as the energy of a ball rolling down a hill.
→Can energy be converted from one form to another?
Yes, energy can be converted from one form to another, such as from potential energy to kinetic energy,
or from thermal energy to mechanical energy. However, the total amount of energy in a system remains
constant, according to the law of conservation of energy.
→How does the law of conservation of energy relate to the first and second laws of
thermodynamics?
The law of conservation of energy is related to the first and second laws of thermodynamics. The first law
of thermodynamics states that energy cannot be created or destroyed, only converted from one form to
another, which is similar to the law of conservation of energy. The second law of thermodynamics states
that the total entropy, or disorder, of a system always increases over time, which has implications for the
efficiency of energy conversion processes.
→Can the law of conservation of energy be proven or is it just a theory?
The law of conservation of energy is a well-established and widely accepted scientific principle that has
been supported by numerous experiments and observations. While it cannot be proven in the absolute
sense, it is considered to be a fundamental principle of physics that has been thoroughly tested and
supported by evidence.
→How does the law of conservation of energy apply to renewable energy sources?
The law of conservation of energy applies to renewable energy sources, such as solar, wind, and
hydropower, in that these energy sources convert the energy from one form to another. For example, solar
panels convert light energy from the sun into electrical energy, while wind turbines convert kinetic energy
from wind into electrical energy. The total amount of energy in the system remains constant, according to
the law of conservation of energy.
Ratio of Potential Differences
Ratio of Potential Differences
Number of Turns Ratio

 In a transformer, we know that there is a primary and a secondary coil. The


two coils do not have the same potential difference; this varies across the two.
 The pd of each coil is determined by the number of turns on each coil. The
higher the number of turns on the coil, the higher the pd in that coil. We can work
out a ratio of turns between the primary and secondary coils, and therefore a ratio
of p.d.’s as well.

The transformer equation:

Where:

 The number of turns on the primary coil, Np


 The number of turns on the secondary coil, Ns
 The potential difference of the primary coil, Vp
 The potential difference of the secondary coil, Vs

Question: A transformer has 400 turns on its primary coil and 20 on its secondary coil. Calculate
the potential difference across the primary coil if the potential difference across the secondary
coil is 12 V.

1. Write out the formula.

Vp / Vs = Np / Ns

2. Write out the numbers we know


Vs = 12V
Np = 400 Turns
Ns = 20 Turns

3. Substitute in the numbers.

Vp / Vs = Np / Ns
Vp / 12 = 400/20
Vp / 12 = 20
Vp = 20 x 12
Vp = 240V

Transformers
Basic Structure of a Transformer
The basic transformer is made up of two coils. These are called the primary coil and
a secondary coil.

As the names suggest, they are simply just wires that have been shaped into coils (solenoids).
The primary and secondary coils of the transformer are wound around an iron core, in a similar
way to the solenoids in earlier tutorials.

Transformers
Using Iron
 Iron can be called an ‘induced magnet’. This means that it becomes magnetic very easily in the
presence of a current. This property makes it perfect as the core of a transformer; it enables us
to magnetise and use the transformer very easily.
 Iron is also very easily demagnetised. Once we stop applying the current, the iron loses most of its
magnetism very quickly which allows us to switch the transformer on and off very easily.

Loudspeakers
Varying Pressure in Sound Waves
 In electrical circuits the current can be varied. This can be converted into pressure variations in
sound waves.
 The current variations are converted into pressure variations through the motor effect. This
means that the motor effect is commonly used in loudspeakers and headphones.

Uses of Moving Coils

One practical application of moving coils is in loudspeakers.

Inside a loudspeaker, there is alternating current which passes through coil which is wrapped
around a magnet. There’s a second wire with a separate current passing through it.

The two magnetic fields created interact, causing force to be exerted. When the current through
the coils reverses, the force exerted by the coils will also reverse.

The coils are connected to a cone which vibrates when the coils exert a force. This vibration
changes when the current alternates.

We can control the vibration of the cone by changing the frequency of the alternating current.
This controls the pitch and volume produced by the loudspeaker.
Loudspeakers
→What is a loudspeaker?
A loudspeaker is a device that converts electrical energy into sound energy. It works by vibrating a
diaphragm or cone to create sound waves that can be heard by the human ear.
→What are the parts of a loudspeaker?
A loudspeaker has several parts, including a diaphragm, a voice coil, a magnet, and a frame. The
diaphragm is the part that vibrates to produce sound waves, and the voice coil is the wire that is attached
to the diaphragm. The magnet creates a magnetic field that interacts with the voice coil to produce
movement, and the frame holds all the parts together.
→How does a loudspeaker work?
When an electrical signal is sent to the voice coil, it creates a magnetic field that interacts with the
magnetic field of the permanent magnet. This interaction causes the voice coil to move back and forth,
which in turn causes the diaphragm to vibrate and produce sound waves.
→What is frequency response?
Frequency response refers to the range of frequencies that a loudspeaker can produce. A loudspeaker with
a wider frequency response can produce a wider range of sounds, which makes it better for listening to
music and other types of audio.
→What is impedance?
Impedance is a measure of the resistance that a loudspeaker has to an electrical signal. A speaker with a
lower impedance will draw more power from an amplifier than a speaker with a higher impedance.
→What is sensitivity?
Sensitivity is a measure of how efficiently a loudspeaker converts electrical energy into sound energy. A
more sensitive loudspeaker will produce a louder sound than a less sensitive loudspeaker with the same
input power.
→What are the different types of loudspeakers?
There are several types of loudspeakers, including cone speakers, dome speakers, horn speakers, and
planar magnetic speakers. Each type has its own advantages and disadvantages and is used for different
applications.
→How do you choose the right loudspeaker?
To choose the right loudspeaker, you need to consider several factors, including the size of the room, the
type of music or audio you will be listening to, and your budget. You should also consider the sensitivity,
impedance, and frequency response of the loudspeaker.
→What are the benefits of using a loudspeaker?
Using a loudspeaker can improve the quality of sound and make it easier to listen to music or other types
of audio. It can also enhance the overall listening experience, making it more enjoyable and engaging.
→What are some common issues with loudspeakers?
Some common issues with loudspeakers include distortion, poor frequency response, and insufficient
power handling. These issues can be caused by a number of factors, including poor design, damage to the
speaker components, and improper installation.

Electric Motors
Rotation of Wires
 Placing coils of wire (also called solenoids) into a magnetic field causes them to rotate. This is
because of the direction of the forces exerted on the coil.
 A common use of rotating coils is inside an electric motor. The coils also only rotate when a current
passes through it so the motor can be switched on and off easily.

Forces Causing Rotation


 Conductors can experience forces. This is often when a current passes through a conductor as this
generates a magnetic field. This magnetic field interacts with a second magnetic field causing a force
to be exerted.
 The forces acting on a coil of wire in a magnetic field it will cause it to rotate. This rotation is used
inside an electric motor.

Fleming’s Left Hand Rule


The Motor Effect
1. A current-carrying wire creates a magnetic field. We can place this wire into another magnetic field.
This causes the two magnetic fields to interact.
2. When they interact, the fields exert forces on each other. The wire exerts a force on the field and the
field exerts a force on the wire. This is known as the motor effect.
3. As it exerts a force, the motor effect can cause the wire to move. The force experienced by the wire
is strongest when it is placed at 90 degrees to the existing magnetic force.

Using Fleming’s Left Hand Rule


You can predict the direction of the force-motion effect from Fleming’s left-hand rule.

 The thumb represents the direction the force acts.


 The first finger represents the direction of the magnetic field.
 The second finger represents the direction of the convention current.
Fleming’s Left Hand
Rule

Factors Affecting Force on Conductor

There are three main factors that affect the size of a force on a conductor:

1. Current: The larger the current, the larger the force.


2. Length: The longer the length of the conductor, the larger the force.
3. Magnetic Flux Density: The higher the density, the more magnetic field lines and the larger the force
on the conductor.

Linking Force and Flux Density


The force experienced by a wire in a magnetic field can be calculated using the equation below.

For exams, you need to be able to remember and use the following equation.
Where:

 force, F, in newtons, N
 magnetic flux density, B, in tesla, T
 current, I, in amperes, A (amp is acceptable for ampere)
 length, l, in metres, m

Question: Calculate the force generated when the magnetic flux density is 5T, the current is 30
000 milliamps and the length of wire is 100cm.

1. Get the correct units.

If you don’t convert correctly, the answer may be out by a factor of 100+
Current should be in amps so 30 000 milliamps = 30 amps
Length should be in metres so 100 cm = 1m

2. Substitute in the numbers.

F=BIl
F = 5 x 30 x 1
F = 150 Newtons

FAQs
→What is Fleming’s Left Hand Rule?
Fleming’s Left Hand Rule is a rule used in physics to determine the direction of the force experienced by
a current-carrying conductor in a magnetic field.
→How does the Left Hand Rule work?
To use the Left Hand Rule, you extend your thumb, first finger, and second finger in a straight line,
perpendicular to each other. Your thumb represents the direction of the magnetic field, your first finger
represents the direction of current flow, and your second finger represents the direction of the force
experienced by the conductor.
→When is Fleming’s Left Hand Rule used?
Fleming’s Left Hand Rule is used in a variety of electrical and electromechanical systems, such as electric
motors, generators, and transformers, to determine the direction of the force experienced by a current-
carrying conductor in a magnetic field.
→Why is the Left Hand Rule important in physics?
The Left Hand Rule is important in physics because it helps to explain how electric currents interact with
magnetic fields, which is the basis of many electrical and electromechanical systems. Understanding this
interaction is crucial for designing and analyzing these systems.
→Can you use the Right Hand Rule instead of the Left Hand Rule?
Yes, there is also a Right Hand Rule that can be used to determine the direction of the force experienced
by a current-carrying conductor in a magnetic field. The Right Hand Rule is simply the mirror image of
the Left Hand Rule and can be used interchangeably, depending on personal preference.
→What is the relationship between the direction of the current and the direction of the
force?
The direction of the force experienced by a current-carrying conductor in a magnetic field is
perpendicular to both the direction of the current and the direction of the magnetic field. The direction of
the force can be determined using either the Left Hand Rule or the Right Hand Rule.
→How does the Left Hand Rule explain the operation of an electric motor?
In an electric motor, a current-carrying conductor is placed in a magnetic field, causing a force to be
experienced by the conductor. The direction of this force can be determined using the Left Hand Rule,
and it causes the conductor to rotate. This rotation is used to drive the operation of the electric motor.
→Can you use the Left Hand Rule to determine the direction of the magnetic field?
No, the Left Hand Rule is used to determine the direction of the force experienced by a current-carrying
conductor in a magnetic field. To determine the direction of the magnetic field, a compass or other
magnetic field sensor can be used.
→How does the Left Hand Rule explain the operation of a generator?
In a generator, a rotating conductor is placed in a magnetic field, causing a current to flow in the
conductor. The direction of this current can be determined using the Left Hand Rule, and it provides the
electrical power output of the generator.
→Can you use the Left Hand Rule to determine the direction of the current in a circuit?
No, the Left Hand Rule is used to determine the direction of the force experienced by a current-carrying
conductor in a magnetic field. To determine the direction of the current in a circuit, you can use Ohm’s
Law, Kirchhoff’s Laws, or other electrical laws and principles.

Electromagnetism
Electric Currents and Magnetic Fields
 Current can flow through a wire. This is simply the flow of charged particles.
 When a current flows through a wire, a magnetic field is produced. The shape of the magnetic field
is like a cylinder.
 The magnetic field strength around a current-carrying wire will vary. The closer you are to the
wire, the stronger the magnetic field. The closer together the field lines are, the stronger the magnetic
field.
 We can use the Right Hand Grip Rule. If we know the direction of the magnetic field around a
current carrying wire, we can find out the direction of current flow through the wire. This can be done
using the Right Hand Grip Rule.
Electromagnetism

Effects of Solenoids
 A solenoid is simply a piece of wire that has been shaped into a coil. Like other
wires, solenoids have the ability to carry current, which means that they are capable of producing a
magnetic field.
 Each coil has a magnetic field around it. All these fields face in the same direction, creating a
uniform and strong magnetic field. As there are many magnetic fields are in close proximity, the
magnetic field is stronger than that around a straight wire.

Magnetic Fields in Solenoids


 The magnetic field is parallel to its axis. Inside a solenoid, the magnetic field runs parallel to its axis.
 The field is different outside the solenoid. Out the solenoid, the field is similar to that of a bar
magnet.
 We can increase the magnetic field strength of a solenoid by including an iron core. This is
because the iron becomes magnetic in the presence of a current. However when the current is removed,
the iron loses its magnetism.
Electromagnetism

Demonstrating Effects of a Current


 We can prove the magnetic effect of currents using wires. This is clearest with electromagnets as
they can be switched on/off by switching the current on/off.
 An electromagnet is very similar to a solenoid with an iron core. An electromagnet is a soft iron
metal core surrounded by coils of insulating copper wire.
 An electric current produces the magnetic field. An electromagnet’s magnetic field is produced by
an electric current flowing through the coil surrounding the metal core.

Electromagnetism

Drawing Magnetic Field Patterns


Electromagnetism

Electromagnetism

Straight Wire

Electromagnetic devices can be represented as circuit diagrams.

Exams require interpretation of simple electromagnetic devices such as the one shown opposite.

The diagram below represents an electronically controlled bell.


Electromagnetism

Magnetic Fields
Defining Magnetic Fields
Exams may ask you to define the term ‘magnetic field’. Here’s a simple definition to answer
those types of questions.

The magnetic field is the region around a magnet where a force acts on another magnet or
magnetic material (iron, steel, cobalt and nickel).

Forces in Magnetic Fields


 Magnets exert forces. We have mentioned this several times before, but it is important to remember
that the poles of a magnet will exert forces. These forces can either be a force of attraction or a force
of repulsion.
 Magnetic materials experience forces. When we place a magnet next to certain materials, they will
experience a force of attraction. These certain materials are known as magnetic materials. They will
always be attracted to magnets.

Strength of Magnetic Fields


 Distance is important. The distance in between the magnet and the magnetic object is very important
when determining the strength of the magnetic field. The closer we hold an object to the magnet, the
stronger the magnetic field will be.
 The field strength is strongest at the poles. As we discussed previously, the magnetic field
is strongest at the poles of the magnet. This means that if an object is placed close to the pole of a
magnet, then it will experience a stronger magnetic field than if it was simply placed to the side.

Magnetic Fields

Direction of Magnetic Fields


 We can find the direction of a magnetic field. We can find the direction of a magnetic field quite
simply by using force. This ‘force’ is the force that will act on the north pole of the magnet from that
point.

Magnetic Fields
 Magnetic field lines run from north to south. If we take a bar magnet (or any permanent magnet), we
can plot the field lines around the magnet. These field lines will denote the shape of the magnetic field.
The important thing to remember is that these field lines will run from the north pole of the magnet to
the south pole of the magnet.

Uniform Magnetic Fields


 Two opposite poles form a uniform magnetic field. Placing the opposite poles of two magnets close
to each other will produce a uniform magnetic field.
 The strength and direction is constant. The strength and direction of the magnetic field will be
the same at any point in the field.

Magnetic Fields

Magnetic Compass
 A magnetic compass has a bar magnet. This magnet has north and south poles. These poles can be
attracted to the magnetic north or magnetic south pole of the Earth.
 Compasses use the Earth’s magnetic field. We can align the magnet to find the direction of
the Earth’s magnetic field. This is because the north pole of the magnet will point towards the Earth’s
magnetic south pole.

Drawing Magnetic Fields


We can draw magnetic fields in one of two ways; by using a bar magnet, or by using a magnetic
compass. We’ll explore these in more detail in this section.

Using a Bar Magnet


1. Place a bar magnet onto a piece of paper and draw around it. Label one end North, and the other
end as South.
2. Place a bar magnet onto a piece of paper and draw around it. The field lines should run away from
the North Pole of the magnet, towards the South Pole, as shown in the image.
Using a Magnetic Compass
1. Place a bar magnet onto a piece of paper and draw around it. Label one end North, and the other
end as South.
2. Place a magnetic compass on the paper. Make sure that the compass is close to the North Pole of the
magnet. On the piece of paper, draw on a small arrow in the direction that the compass is pointing.
3. Move the magnetic compass away from the North Pole, towards the South pole. Continue to mark
the arrows onto the piece of paper.
4. You should end up with a series of arrows running from North-South. Repeat this at various points
around the magnet, until you get a diagram like the one shown.

Magnetic Fields

Behaviour of A Magnetic Compass


 Magnetic compasses rely on the Earth. Magnetic compasses ‘line up’ in the direction of the
magnetic field of the Earth.
 The Earth is magnetic. The Earth is magnetic as it affects magnetic compasses and produces
a magnetic field.

→What is a magnetic field?


A magnetic field is an invisible force field that is produced by magnets or moving charges. It can be
visualized using magnetic field lines, which indicate the direction and strength of the field.
→What are the properties of magnetic fields?
Magnetic fields have several properties, including direction, strength, and polarity. They can be attracted
or repelled by other magnetic fields and can influence the motion of charged particles.
→What is the relationship between electricity and magnetism?
Electricity and magnetism are closely related, as changing electric fields produce magnetic fields, and
changing magnetic fields produce electric fields. This relationship is known as electromagnetism and is
the basis for many modern technologies.
→What is a solenoid?
A solenoid is a coil of wire that produces a magnetic field when an electric current is passed through it.
Solenoids are used in many devices, including speakers, motors, and electromagnetic locks.
→What is an electromagnet?
An electromagnet is a type of magnet that is created by passing an electric current through a coil of wire.
The strength of the electromagnet can be controlled by changing the amount of current that is flowing
through the coil.
→What is the Earth’s magnetic field?
The Earth has a magnetic field that is produced by the movement of molten iron in its core. This magnetic
field helps to protect the Earth from harmful solar radiation and is responsible for the phenomenon of
auroras.
→How do magnetic fields affect charged particles?
Magnetic fields can affect the motion of charged particles, causing them to move in a curved path. This
effect is used in devices such as cathode ray tubes, particle accelerators, and MRI machines.
→What is a magnetic domain?
A magnetic domain is a region of a magnetic material in which the magnetic fields of the atoms are all
aligned in the same direction. This alignment creates a strong magnetic field within the domain.
→How are magnetic fields used in technology?
Magnetic fields are used in many technologies, including generators, motors, speakers, and hard drives.
They are also used in medical imaging, such as MRI machines, and in navigation systems, such as
compasses and GPS devices.
→How can we protect ourselves from the harmful effects of magnetic fields?
Some people are concerned about the potential harmful effects of magnetic fields on human health. To
protect yourself, you can limit your exposure to magnetic fields by avoiding high-voltage power lines,
limiting your use of electronic devices, and using shielding materials. However, the scientific community
has not yet established a clear link between magnetic fields and adverse health effects.

Background Radiation
Background Radiation Sources
 Background radiation is all around us. In the air around us, there is always a low level
of background radiation. This radiation will comes from various sources.
 Background radiation can come from natural sources. Rocks are a natural source of background
radiation, whilst cosmic rays can bring background radiation from space. Food and drink can even be
natural sources of background radiation. The unstable isotopes in these sources will lead to the
background radiation.
 Background radiation can come from man-made sources. Sometimes, the background radiation can
come from a man-made source. These sources tend to be from nuclear weapons testing and nuclear
accidents. Although these are very dangerous sources of unstable isotopes, they only make up a small
amount of all background radiation. They can also come from medical equipment, such as X-rays.

Measuring Radiation
Measuring Radiation
Detecting Radiation

 We can detect radiation using photographic film. As the photographic


film absorbs radiation, it becomes darker.
 Film badges monitor radiation. You will often see people who work with or
near radiation wearing film badges. These badges monitor the levels of radiation
absorbed, so workers can regularly check and control their exposure to radiation.
 Badges contain different materials. Inside the badge, you will find different
materials which absorb different amounts of radiation. The radiation therefore has
to penetrate through the different materials to reach the film.

Measuring Radiation

Measuring Background Radiation

 We can measure background radiation. Using a Geiger-Muller tube, we can


measure the level of background radiation in the air. The Geiger-Muller tube
should be placed away from any known sources of radiation, so that we can get an
accurate measurement for background radiation.
 The unit of radiation is the sievert. Sieverts are the units of the radiation. The
symbol for sievert is Sv. Like most other units, sieverts can come in different
factors, such as millisieverts (mSv).
 Millisieverts make up sieverts. 1000 millisieverts make up one sievert. Since
background radiation is at such a low level, you will commonly see the unit mSV
rather than the unit Sv.

Uses of Nuclear Radiation


Medical Uses
 Gamma rays are used to treat cancer. Patients with cancer are often treated with a form of gamma
rays, called radiotherapy. A strong beam of gamma rays is aimed at the cancerous tissue. The
cancerous tissue is killed due to the radiation exposure.

Uses of Nuclear Radiation


 Gamma isotopes are used as tracers. A different form of gamma rays can be used as a tracer.
Patients can be given an injection or a drink containing a gamma isotope. Using a special gamma
camera, the isotope can be traced as it moves through the body. For example in Positron Emission
Tomography (PET) scans. The isotopes have very short half-lives, so need to be produced nearby.
They are manufactured in the hospital and injected to the patient’s blood as soon as possible.
 Iodine is used in the thyroid gland. A radioactive isotope of iodine can be used to check the function
of the thyroid gland. The radioactive isotope will be absorbed and traced as it moves into the thyroid
gland.
 Sterilisation of equipment. This is common with surgical equipment. Sterilisation can be carried out
without high temperatures, however it may not kill all the bacteria on an object.

Other uses
 Household fire alarms. During a fire, the smoke absorbs alpha radiation. This then ionises the air
inside a smoke detector, triggering the fire alarm to go on.

Uses of Nuclear Radiation


 Irradiating food. Food can be exposed to a radioactive source to preserve it. Usually gamma rays are
used, which destroy any bacteria on the food without affecting it. This can be seen on fruit and
vegetables in supermarkets.
 Tracing and gauging thickness. Thicker materials absorb more radiation. Therefore a detector would
absorb less radiation. This can be used in industry for detectors which monitor and control
the thickness of different materials. Signals are sent to the machines to adjust the thickness of the
material accordingly

FAQs
→What is nuclear radiation and how does it differ from other forms of radiation?
Nuclear radiation is a type of radiation that is produced by the nucleus of an atom. It differs from other
forms of radiation, such as light and heat, in that it is more energetic and has the ability to penetrate
materials. Nuclear radiation can also be harmful to living organisms and the environment.
→What are some common uses of nuclear radiation in everyday life?
Nuclear radiation is used in many different applications in everyday life, including medical imaging,
cancer treatment, food preservation, and smoke detectors. In medical imaging, nuclear radiation is used to
produce images of the inside of the body, such as x-rays or CT scans. In cancer treatment, nuclear
radiation is used to kill cancer cells. Nuclear radiation is also used to preserve food by killing bacteria and
other microorganisms, and to detect smoke in smoke detectors.
→How is nuclear radiation used in medical applications?
Nuclear radiation is used in medical applications to produce images of the inside of the body, to treat
cancer, and to sterilize medical equipment. For example, x-rays, CT scans, and PET scans all use nuclear
radiation to produce images of the inside of the body. In cancer treatment, nuclear radiation is used to kill
cancer cells and to shrink tumors. Nuclear radiation is also used to sterilize medical equipment to prevent
the spread of infection.
→How does the use of nuclear radiation in medicine compare to other medical
technologies?
The use of nuclear radiation in medicine is just one of many medical technologies available. It is a
valuable tool for medical imaging and cancer treatment, but it is not the only option. Other technologies,
such as ultrasound, magnetic resonance imaging (MRI), and computed tomography (CT) scans, also play
an important role in medical diagnosis and treatment. The choice of which technology to use depends on
the specific needs of the patient and the medical situation.
→What are some of the potential risks and benefits of using nuclear radiation in medicine?
One potential risk of using nuclear radiation in medicine is exposure to harmful levels of radiation. This
can cause long-term health effects, such as an increased risk of cancer and other diseases. On the other
hand, nuclear radiation can also provide many benefits in medicine, such as improved accuracy in
medical imaging and cancer treatment, and improved patient outcomes.
→How is nuclear radiation used in other industries, such as food and energy production?
Nuclear radiation is also used in other industries, such as food and energy production. In the food
industry, nuclear radiation is used to preserve food by killing bacteria and other microorganisms. In
energy production, nuclear radiation is used in nuclear power plants to produce electricity.
→Why is it important to study the uses of nuclear radiation in physics?
The study of the uses of nuclear radiation in physics is important because it helps us understand the
properties and behavior of nuclear radiation and how it can be used in different applications. This
knowledge is essential for advancing our understanding of the fundamental principles of physics and for
developing new technologies and applications in the future.

Risks of Radiation
Nuclear Radiation
Perceived Risks of Radiation
 Radiotherapy has lots of side effects. Patients sometimes experience negative side effects of
radiotherapy. A common side effect is radiation sickness, where patients get ill due to the
radiotherapy.
 Benefits can outweigh risks. Although radiation sickness can be seen as a negative risk of
radiotherapy, there are often more benefits than risks with this method of treatment. Radiotherapy
can kill cancer cells and remove tumours, which is a huge benefit to patients.
 Perceived risk depends on the patient. Although there are facts and figures to outline risks and
benefits of treatment, ultimately only the patient can decide how ‘risky’ a treatment will be for them.
Therefore, perceived risk comes down to each individual patient.

Disposing Nuclear Waste

Radioactive waste is extremely reactive, therefore it must be stored or disposed with caution.

Since the waste could have a long half-life, it must be stored for long periods of time. For
example in containers, however it is important they do not degrade as leaks could occur.

They also need to be stored in safe locations, away from water (as this could cause
contamination) and in areas where natural disasters, such as earthquakes, are less likely to occur.
Risks of Radiation

Evaluation of Uses
Evaluating the Uses of Nuclear Radiation
Evaluation of Medical Uses
 Radiotherapy can cause tissue damage. Doctors must be careful when giving radiotherapy to
patients, since the gamma rays can kill normal tissue too. Therefore, they need to strike a balance
between killing the cancer cells and not destroying too many normal cells.
 Tissue damage can lead to mutations. If the radiation damages the DNA within a cell, the cell can
rapidly divide. This abnormal behaviour is called a mutation, where the cell now becomes cancerous.
 Gamma rays are good for exploration. Doctors are able to use gamma rays to explore internal organs
due to their weak ionising power. The main benefit of these tracers is that they allow life threatening
conditions to be diagnosed, whilst not ionising too much tissue.

Internal and External Radiation


Tumours can be treated using either internal (inside the body) or external (outside the body)
radiation. The level of danger varies depending on the type of radiation used, summarised in the
table You need to be able to compare below.
Evaluation of Uses

Nuclear Fission
Splitting Nuclei
 We can split a nucleus. Through the process of nuclear fission, we can split a large, unstable nucleus
into 2 smaller particles. For example Uranium-235, which is a naturally occurring isotope of Uranium.
This is made possible through the use of a nuclear reaction.
 Nuclei undergo induced fission. Nuclei don’t often split up on their own; instead, they have to
undergo induced fission. During this process, they absorb a neutron, which leads to fission.
 Spontaneous fission is rare. Sometimes, a nucleus doesn’t have to absorb a neutron to undergo fission.
This is called spontaneous fission, but this is very rare.

Process of Nuclear Fission


 The nucleus absorbs a neutron. In most fission reactions, the unstable nucleus will absorb a neutron
as the first step.
 The nucleus splits into 2. Once the neutron has been absorbed, the nucleus will split into 2 smaller
‘daughter‘ nuclei. Energy is released through the fission reaction.
 Neutrons and gamma rays are emitted. The nucleus will also emit 2 or 3 high energy neutron when
the fission reaction is happening. As well as 2 or 3 neutrons, gamma rays are also emitted.
Nuclear Fission

Products of Nuclear Fission


 Fission products have KE. The neutrons and 2 smaller nuclei released in a fission reaction are highly
radioactive and will have kinetic energy. The KE allows the fission products to move away from the
original reaction site.
 Gamma rays carry excess energy. Any energy that is not absorbed by the products of the reaction is
given to the gamma rays. This means that the gamma rays will move away from the reaction site as
well, often at very high speeds.
 Neutrons can start a chain reaction. The neutrons released from a fission reaction will have kinetic
energy. Therefore, the neutrons can move around and become absorbed by another large, unstable
nucleus. This will lead to a second fission reaction, resulting in a chain reaction.

Chain Reactions
Controlling the Chain Reaction
 Chain reactions can be dangerous. It is very difficult to stop a chain reaction, meaning it can
become dangerous very quickly. Many neutrons are produced by each fission reaction, leading to
multiple fission reactions.
 Nuclear reactors use control rods and moderators. Inside a nuclear reactor, there are a controlled
series of fission reactions occurring. We do this by using control rods inside the reactor. Usually water
or carbon are used as moderators, also acting as a coolant.
 Control rods absorb neutrons. By placing control rods into the reactor, we can absorb excess
neutrons. We know that neutrons induce fission reactions, so by absorbing some of them, we are
limiting the number of fission reactions that occur.
 Nuclear weapons have uncontrolled reactions. Inside a nuclear weapon, there are uncontrolled
nuclear fission reactions happening. This means that there is an uncontrolled chain reaction, which
ultimately leads to an explosion.
 The reaction must be shielded. A large amount of radiation is produced by nuclear fission, which can
be very hazardous. Therefore the reaction should be shielded, absorbing any radiation and reducing the
risk of harm.
Representing Nuclear Fission
Below, we can see a representation of nuclear fission. You may be asked to draw a diagram like
this in an exam, so here’s how to do it:

1. Draw a neutron. This is essentially a small circle. You can label it ’n’.
2. Draw a large nucleus. As you can see in the diagram below, the nucleus must be made up of smaller
particles.
3. Draw 2 more nuclei. Since this is a fission reaction, we need to draw the 2 neutrons that are produced.
4. Draw 2 or 3 neutrons. Fission reactions produce more neutrons, which we need to represent on our
diagram as small circles again.
5. Draw more fission reactions. From one of the neutrons, draw an arrow to another nuclei. This is the
continuation of the chain reaction.

The end product should look similar to the diagram below:

Chain Reactions

Generation of Electricity
The chain reaction can be used to provide nuclear power in power stations, to
generate electricity.

1. Nuclear energy. Usually, uranium and plutonium are used as nuclear fuels in the chain reaction.
2. Thermal energy. As heat is produced through the chain reaction, water is heated to produce steam in a
boiler.
3. Kinetic energy. The steam then turns a turbine, which turns a generator to produce electricity.
Chain Reactions

Nuclear Fusion
Joining Nuclei
 Nuclear fusion joins nuclei together. The process of nuclear fusion joins nuclei together. This is
essentially the opposite of nuclear fission; 2 small nuclei will be joined together to form a larger
nucleus.
 The end product of nuclear fusion is heavy. Since we are combining 2 small nuclei to make a larger
nucleus, the end product of the reaction is heavy. This fusion leads to a lot of energy being produced.
 Mass can be converted into energy. We have just mentioned that fusion reactions can produce lots of
energy. When the two small nuclei join together, some mass is lost. This ‘lost’ mass gets converted
into energy, which is released as radiation.

Conditions for Nuclear Fusion


A high temperature and pressure is needed for nuclear fusion to occur.

A high temperature provides enough energy to overcome the electrostatic repulsion, found
between the positively charged nuclei of each isotope.

The same applies to pressure; high pressures are needed to overcome the electrostatic repulsion.
The specific conditions required make it very difficult to maintain a practical and economically
friendly power station.

Fusion in Stars
Fusion in Stars
Nuclear fusion is the energy source for stars.

A star is essentially a ball made of hydrogen gas. When hydrogen nuclei fuse together in a star,
by the process of nuclear fusion helium nuclei are formed. A neutron and energy are also
released in this reaction.

In larger stars, this process can repeat where helium nuclei continue to fuse, forming heavier
elements.

Associated Hazards and Precautions


Hazards and Precautions
Associated Hazards

 Risks depend on the source. For some sources, irradiation can pose risks,
whereas for other sources, contamination can pose risks. Irradiation is linked to
beta and gamma sources, whilst contamination is linked to alpha sources.
 Beta and gamma sources can get into the body. Through
irradiation, beta and gamma sources can use their ability to travel long distances
and get inside the body. Alpha sources are unable to travel very far and so they
cannot get into the body through irradiation.
 Alpha sources work in small areas. If an alpha source is in a closed area, then it
can cause a lot of harm by contamination. It is very strongly ionising, so it can
contaminate a lot of particles in a small space, for example inside the body.

Associated Hazards and Precautions

Precautions Against Hazards

 Irradiation can be prevented. We can prevent unwanted irradiation quite


easily. To keep alpha and beta sources from irradiating objects, we can limit
exposure. Workers can limit and monitor their exposure using badges containing
photographic films.
 Sources can be removed. By placing the sources into appropriate containers, we
can keep radiation sources away from objects. We can also place them into
separate rooms. Irradiation will stop as soon as the source is removed, whereas
contamination can be difficult to remove completely.
 Protective clothing can be worn. As well as placing sources appropriately, we
can wear clothing to protect us from radiation. We can wear gloves so that
radioactive particles don’t stick to our skin, or use forceps to handle radioactive
substances instead. We can even wear masks so we don’t accidentally breathe in
any particles.
Associated Hazards and Precautions

Studying Hazards

 Scientists study hazards. We can study hazards of radiation to find out its
effects on humans. This is particularly important, since we need to know how to
protect ourselves from radiation.
 Studies can be published. Scientists can publish the results of their studies to the
public. This allows other scientists to read their work, so that the findings can be
backed up or improved on for the future. Studies are often checked by a peer
review for this to happen.

You might also like